Sie sind auf Seite 1von 104

AIIMS Jodhpur & Rishikesh 2017

1. Kit supplied at subcentre called as


a. Kit E b. Kit B
c.Kit K d. Kit M
Ans : b

2. In polio vaccine vial, inner square of VVM is lighter than outer circle. It means
a. Discard the vaccine
b. Shake well before use
c. Bring the vaccine to the room temperature before using it
d. Use the vaccine
Ans : d

3. A patient in the emergency department has an endotracheal tube in place and is


on the ventilator. His relative comes running to you telling you that the patient has
pulled the tube out of his mouth. What would be your next step?
a. Apply face mask with high flow oxygen
b. Sit the patient up and consider chest physiotherapy to get the secretions out
c. Reinsert the tube immediately under sedation
d. Pull the tube out completely, bag and mask patient and assess sponataneous
breathing efforts
Ans : d

4. While administer eardrop infant, the nurse should pull the pinna of the ear
a. Downward & upward b. Upward & backward
c. Downward & backward d. Backward & upward
Ans : c

5. During blood transfusion, the most important nursing responsibility is


a. Draw a sample from the patient before each unit is transfused
b. Run the blood at a slower rate during the first few minutes of the transfusion
c. Warm the blood product to body temperature to prevent chilling
d. Maintain patency of the IV catheter with dextrose solution
Ans : b

6. Which among the following vaccine is freezer


a. BCG b. Measles
c.DTP d. TT
Ans : b

7. Temperature exposure indicator of a vaccine is


a. VVM b. VAP
c.VMV d. VTI
Ans : a

8. Site for subcutaneous injection is


a. Scapula b. Anterior thigh
c.Lower abdomen d. Anteior upper arm
Ans : a,b,c

9. A patient on antipsychotic medication developed rolling of eyes upward. This


indicative of
a. Therapeutic action
b. Drug has not reached its therapeutic level in blood
c. Extrapyramidal symptoms
d. Myasthenia gravis
Ans : c

10. A relative of a schizophrenic patient informed nurse that patient is having suicidal
ideation. All of the following nursing action is appropriate in this situation EXCEPT
a. Inform the Doctor regarding the situation
b. Do not ask patient direct question regarding suicidal ideation
c. Increase the environmental safety
d. Continuous observation of the patient
Ans : b

11. A nurse in the labor room performing vaginal assessment on a pregnant client in
labor and notes the presence of the umbilical cord protruding from uterus. What
would be the initial nursing action
a. Gently push the cord into vagina
b. Place the client in Trendelenburg’s position
c. Perform vaginal exam to not the presentation
d. Notify that the client will be transported immediately to delivery room
Ans : b

12. The term used to describe the discharges from the uterus during the puerperium
and has an alkaline reaction is
a. Vaginal discharge b. Puerperal discharges
c.Amniotic secretions d. Lochia
Ans : d

13. Look at the picture and identify


picture
a. Laryngeal mask airway b. oxygen mask
c.Endotracheal tube d. Single lumen airway
Ans : a

14. The normal rate of cervical dilation in the active phase of first stage of labor in a
primigravida is
a. 0.5 cm/hr b. 1 cm /hr
c.1.5 cm/hr d. 2 cm /hr
Ans : b

15. All of the following are mode stabilizing agents, EXCEPT


a. Lithium carbonate b. Fluoxetine
c.Valproic acid d. Carbamazepine
Ans : b

16. A phenomenon during therapeutic nurse client relationship when the client
unconsciously attributes ( or transfers ) to the nurse feelings and behavioural
predispositions formed towards a person from his or her past is
a. Resistance b. Boundary violation
c.Transference d. Counter transference
Ans : c

17. False sensory perception not associated with real external stimuli is called
a. Concrete thinking b. Hallucination
c.Delusion d. Illusion
Ans : b

18. A client with schizophrenia has been started on medication therapy with
clozapine. The nurse would assess the result of which laboratory study to monitor
for adverse effect of this medication
a. Liver function studies b. Blood glucose level
c.White blood cell count d. Platelet count
Ans : c
19. What should be added to normal saline which is connected to arterial line
measuring invasive blood pressure?
a. Calcium b. Potassium
c.Protamine d. Heparin
Ans : d

20. A nurse is monitoring manometer for CVP value from a central line inserted via
the brachial vein. CVP line inserted in a patient in ICU on face mask with oxygen.
For correct reading she should record the value at
a. She should disconnect the manometer just before reading
b. End of expiration
c. End of inspiration
d. All the above
Ans : b

21. Each community development block should have


a. One community health center
b. Two community health center
c. Three community health center
d. Four community health center
Ans : a

22. The doctor has ordered 500 ml 5% dextrose to be administered for 5 hours. It
should be administered using a burette set. The nurse has to set the drops per
minute to deliver the appropriate amount. Calculate the flow rate
a. 100 microdrops per minute
b. 80 microdrops per minute
c. 100 macrodrops per minute
d. 80 macrodrops per minute
Ans : a

23. All of the following are true for women with breech presentation at term seeking
vaginal breech delivery, EXCEPT
a. Diagnosing breech presentation for the first time in labour is a
contraindication to vaginal breech delivery
b. Continuous electronic fetal heart rate monitoring should be offered to women
with breech presentation in labour
c. Vaginal breech delivery should be conducted in presence of a person skilled in
the conduct of labour with breech presentation
d. Vaginal breech delivery should take place in a hospital with facilities for
emergency caesarean section
Ans : c

24. While assessing a patient’s cranial nerves, the nurse asks the patient to raise the
eyebrows, smile and show the teeth to assess which cranial nerve
a. Vagus b. Facial
c.Olfactory d. Optic
Ans : b

25. A women using diaphragm for contraception should be instructed to leave it in


place for at least how long after intercoutse?
a. 6 hours b. 12 hours
c.1 hour d. 28 hours
Ans : a

26. During a dressing change, inspection of the wound reveals what appears to be
reddish pink tissue in the wound. The nurse interprets this as
a. Exudates b. Granulation tissue
c.Eschar d. A sign of infection
Ans : b

27. Look at the picture and identify the procedure usually done in a labor room
picture
a. NST b. Color Doppler
c.ECG d. Amniocentesis
Ans : a

28. Order is 180 mg of cefotaxime intravenously. Available form is 500mg/5ml. what


is the amount of medication to be administered using a 2ml syringe with 10
divisions per ml
a. 8 divisions b. 18 divisions
c.12 divisions d. 108 divisions
Ans : b

29. A 40 –year –old unconscious patient is brought to the casualty following road
traffic crash. His blood pressure was 80/60 mm of Hg. What will you do first
a. Protect airway b. Start vasopressors
c.CT scan Head d. Infuse 2 liters of crystalloids
Ans : a
30. Which of the following is considered the fifth vital sign in emergency department
a. Motor response b. Response to pain
c.Verbal response d. Pupillary reaction to light
Ans : d

31. A resident has given a top of epidural infusion of 3 mg morphine with 10 ml of


0.1% ropivacaine. Which of the following symptom requires immediate action
a. Bladder bowels incontinence b. Confusion
c.Fast pounding pulse d. Hallucinations
Ans : b

32. Neural tube defects can be prevented by


a. Supplementation with hydantoin
b. Supplementation with vitamin B complex
c. Maternal folate Supplementation
d. Prophylaxis with valproic acid
Ans : c

33. During ward rounds, you find that a 24 years old patient with mania is very
abusive and aggressive, there is imminent danger of client becoming physically
assaultive. What you should NOT do
a. Avoid reassuring touching of the patient
b. Confront him
c. Keep a safe distance from him
d. Use of clear language
Ans : b

34. A patient with a diagnosis of urosepsis has been admitted to the ward. You have
been asked to start noradrenaline for hypotension. What is the normal route of
administration
a. Wide bore cannula in the posterior tibial vein
b. Peripheral cannula
c. Wide bore cannula placed in the brachial vein
d. Central venous line in the subclavian vein
Ans : b

35. The nurse is caring for a child with sickle cell anemia. To prevent thrombus
formation in capillaries, as well as other problems from stasis and clotting of blood
in the sickling process, the nurse should
a. Administer oxygen
b. Increase fluids by mouth and use a humidifier
c. Encourage the child to maintain bed rest
d. Administer prescribed anticoagulants
Ans : b

36. A hypothermic neonate is airlifted from snowing gulmarg and is admitted in NICU.
the attending nurse is asked to record the core temperature by the attending
doctor while he is busy resuscitating another child. Which area would be most
accurate of the core temperature
a. Axillary b. Forehead skin
c.Oral temperature d. Tympanic membrane
Ans : d

37. A patient have difficulty in smelling, nurse assess for which cranial nerve
a. Cranial nerve I b. Cranial nerve II
c. Cranial nerve III d. Cranial nerve IV
Ans : a

38. Nursing care in a child with extrophy of bladder is ……………


a. Dressing with normal saline b. dressing with petroleum jelly
c.Cover with clean plastic wrap d. Catheterize the child
Ans : c

39. In which stage there are formation of blisters


a. Stage I b. Stage II
c.Stage III d. Stage IV
Ans : b

40. Common position in a child with tetralogy of Fallot


a. Squatting b. Prone
c.Supine d. Side lying
Ans : a

41. Commonest site used for IM injection in infant is


a. Vastus latrelis b. Gluteal muscle
c.Deltoid d. Biceps
Ans : a

42. Angle used for giving IM injection is


a. 45 degree b. 90 degree
c.30 degree d. 120 degree
Ans : b

43. All are types of hemolytic anemia EXCEPT


a. G6PD deficiency b. Thalassemia
c. Spirosis d. Sickle cell anemia
Ans : a

44. Look at picture and identify


picture

a. Episiotomy scissors b. Allies forceps


c.Babcob d. Tooth forceps
Ans : a

45. Doctor ordered 0.35 mg and available drug is 0.25 mg per ml. what is the amount
of drug should be given
a. 1.4 ml b. 0.4 ml
c.1.8 ml d. 2.4 ml
Ans : a

46. Normal pH range of blood is


a. 7.35-7.45 b. 7.25-7.35
c.7.45-7.55 d. 7.15-7.25
Ans : a

47. 0.45% normal saline is the


a. Hypotonic b. Isotonic
c.Hypertonic d. Colloid
Ans : a

48. A patient complaining nausea after having food. It is which type of data
a. Subjective b. Objective
c.Personal d. None of them
Ans: a

49. A farmer came in emergency room with BP 80/60 mm of Hg and history of


vomiting. Symptoms indicating
a. Severe dehydration b. Anaphylactic shock
c.Mild dehydration d. Renal failure
Ans : a

50. First day of menstrual cycle is 14 august. The EDD according to Negeles is
a. 21 may b. 14 june
c.21 june d. 14 may
Ans : a

51. A unconscious child came to you with dehydration, the nurse do first
a. Start IV infusion of RL b. Reassure the parents
c.Give ORS to the child d. Collect history form parents
Ans : a

52. You are giving dextrose 50% to a patient IV, all are complication, except
a. Edema b. Renal failure
c.Hyperglycemia d. Thrombophlebitis
Ans : b

53. We will assess which lab value in renal failure patient


a. Serum creatinine b. Serum calcium
c.Serum magnesium d. Serum chlorine
Ans : a

54. In the following is not a complication of giving potassium chloride intravenous


a. Phlebitis b. Hyperkalemia
c.Numbness d. Constipation
Ans : d

55. A client is receiving IV fluids and develops tenderness, erythema and pain at the
site. The nurse suspects which of the following
a. Fluid overload b. Sepsis
c.Phlebitis d. Infiltration
Ans : c

56. A patient receiving continuous salbutamol nebulization therapy. Nurse assess for
a. S. calcium b. S. potassium
c.S. magnesium d. S. sodium
Ans : b
57. You will use which solution for spillage of blood on the floor
a. Chloride mixed solution b. Phenol mixed
c.Cidex d. Formaline
Ans : a

58. Which is the long acting insulin


a. Glargine b. NPH
c.Regular d. Lispro
Ans : a

59. Cation present mostly in plasma is


a. Sodium b. Potassium
c.Calcium d. Magnesium
Ans : a

60. The diameter (size) of show picture is starts from


picture

a. 5 mm b. 4.7 mm
c.4.3mm d. 5.3 mm
Ans : a

61. A 60 kg patient ordered IV Amikacin 25 mg/kg body weight/per day. After


dividing in two parts the accurate single dose is
a. 750mg b. 650 mg
c.700 mg d. 450 mg
Ans :a

62. In Chest tube drainage system there is fluctuation in water seal chamber with
respiratory efforts. Nurse finds it as
a. Kinking of chest tube b. Normal process
c.Air leaking d. None of them
Ans : b

63. There is no oscillation seen in chest tube drainage system inserted for
pneumothorax. The nurse understands that
a. Tube is blocked b. It is normal
c.Air leaking d. None of the above
Ans : a
64. A patient receive highest oxygen by
a. Mask with reservoir b. Venturi mask
c.Nasal cannula d. Simple mask
Ans : a

65. Pepsinogen secretes oxygen by


a. Chief cells b. Mast cells
c.Beta cells d. Parietal cells
Ans :a

66. Normal bone cell is known as


a. Osteocyte b. Osteoblast
c.Osteoclast d. Monocyte
Ans : a

67. A patient is admitted in ward with prior MI attack. He complains for chest pain,
first action of nurse is
a. Provide nitro – glycerine S/L b. Get an ECG done
c.Order trop t d. Order an echo
Ans : a

68. A nurse will keep in isolation of a patient with bacterial meningitis for
a. 24 hr after the initiating of antibiotic therapy
b. 12 hr after the initiating of antibiotic therapy
c. Till culture negative
d. After 12 hour of antibiotic given
Ans : a

69. A patient complaining visual disturbance, nurse knows it is the effect of which
drug
a. Etambutol b. Pyrazinamide
c.Rifampicin d. Lithium
Ans : a

70. Precautions used when caring for a rubella patient is


a. Droplet b. Contact
c.Universal d. Hand washing
Ans : a
71. A women came with complaining of recurrent uterine bleeding, nurse action
involve
a. Cryotherapy b. Prepare pap smear
c.Anti – haemorrhagic drugs d. Reassure the client
Ans : b

72. A patient shifted in operation theatre. Anesthetist instructs to place ECG


monitoring and a 3 lead ECG available. What will you do
a. Observe with II lead b. Cancel the operation
c.Look for a 12 lead ECG d. Shift patient to ward
Ans : a

73. Effect of vasodilator includes


a. Hypotension b. Hypertension
c.Diuresis d. Sweating
Ans : a

74. Osmolarity of normal saline is


a. 134mEq/L B. 144 mEq/L
c.154mEq/L d. 164 mEq/L
Ans : c

75. A unconscious patient brought by their relatives, your priority should be


a. Remove secretions and clean airway
b. Provide oxygen
c. Give adrenaline
d. Call the physician
Ans : a

76. The drug used to prevent aspiration and decrease the secretions is
a. Atropine b. Adrenaline
c.Noradrenaline d. Dopamine
Ans : a

77. A patient had sudden reaction after administering a particular drug, this is known
as………
a. Anaphylactic b. Septic
c.Side effect d. None
Ans: a
78. To increase the effect of local anesthesia use of
a. Epinephrine b. Norepinephrine
c.Atropine d. Dobutamine
Ans : a

79. How a nurse assess the proper functioning of laryngeal nerve after thyroidectomy
a. Tell to speak words b. Throat swab for gag reflex
c.Provide oral fluids d. Check motor response
Ans : a

80. Adrenaline used for dental anesthesia is


a. 1:100000 b. 1:10000
c.1:200000 d. 1:100
Ans : c

81. Look at the picture and identify the grip


Picture
a. Fundal grip b. Lateral grip
c.Abdominal grip d. Umbilical grip
Ans : a

82. Commonly preferred artery for ABG assessment in an OPD setting is ………
a. Radial b. Brachial
c.Femoral d.Carotid
Ans : a

83. PH of vagina becomes acidic due to


a.Bartholin gland b. Sebaceous gland
c.Doderlin bacillus d. E.coli
Ans : c

84. Station in OG is related to


a. Ischial spine b. Illac spine
c.Coccyx d. Cervix
Ans : a

85. Normal color of amniotic fluid is


a. Clear b. Dark brown
c.Greenish d. Reddish
Ans : a
86. Most reliable sing to assess fetal condition is
a. Decreased movement of fetus
b. FHR below 100/min
c. FHR above 160/min
d. Amount of amniotic fluid
Ans : a

87. Maximum dose of atropine can be give is


a. 1 mg b. 3 mg
c.5 mg d. 10 mg
Ans : b

88. Patient had reaction after blood transfusion. The first action of nurse is
a. Stop the transfusion immediately
b. Call to the doctor
c. Monitor vital sings
d. Slow the rate of transfusion
Ans : a

89. A nurse caring patient which receiving TPN in ICU, nurses action should be
a. Discard the TPN bag after 24 hours even some fluids in the bag
b. Continue start TPN
c. Stop the TPN for some time
d. All of the above
Ans : a

90. Edema is due to all, EXCEPT


a. Increased hydrostatic pressure
b. Decrease hydrostatic pressure
c. Reduced oncotic pressure within blood vessels
d. Increased blood vessel wall permeability
Ans : b

91. Antidote of MgSO4 is


a. Calcium gluconate b. Potassium chloride
c.Potamine sulphate d. Naloxen
Ans : a

92. Which plane divides the body in anterior and posterior


a. Frontal b. Sagittal
c.Parasagittal d. Transverse
Ans : a

93. Which is true with ‘Kangaroo mother care’


a. Skin to skin contact, breastfeeding ,prevention from infection
b. Skin to skin contact, breast feeding, early discharge
c. Skin to skin contact, early discharge, prevention from infection
d. Early discharge, breast feeding, prevention from infection
Ans : a

94. Inferiority V/S Guilt shown in the age of


a. 0-1 year b. 1-3 year
c.3-6 year d. 6-12 year
Ans : c

95. Universal recipient for blood is


a. O+ve b. O- ve
c.AB+ve d. AB –ve
Ans : c

96. According to Erickson stages of development 4 year old child comes under
a. Preschooler b. Schooler
c.Toddler d. Infant
Ans : a

97. Worker in ICDS programme is


a. Anganwadi b. Village health guide
c.Health supervisor d. MPW
Ans : a

98. A nurse keep in mind while giving heparin injection to a patient


a. Don’t aspirate & don’t massage
b. Do aspirate & don’t massage
c. Don’t aspirate & do massage
d. Do aspirate & do massage
Ans : a

99. Method used to give Haloperidol injection


a. Z track b. M track
c.N track d. W track
Ans : a

100. A child suspected to have pin worm, to confirm the nurse should tell the mother to
a. Bring 3 stool sample and observe with eyes
b. Bring 3 stool sample and send for culture
c. Inspection of perennial area
d. Assess the hygienic status of child
Ans : b

101. A nurse teaches the client to collect urine specimen for bacterial infection includes
a. Collect midstream of early void
b. Collect specimen at any time
c. Collect specimen after first void
d. 24 hour urine collection
Ans : a
102. A child is not giving his play material to another. This type of play is
a. Parallel play b. Group play
c.Social play d. All of the above
Ans : a

103. The recorded BP would be


Picture
a. Higher diastolic but lower systolic BP
b. Higher than actual BP
c. Same as actual BP
d. Lower than actual BP
Ans : c

104. Hearth sounds are produce by


a. Closure of heart valves b. Contraction of atrium
c.Contraction of ventricles d. SA node
Ans : a

105. Common cause of heart valve stenosis in India is


a. Rheumatic fever b. Cardiomegaly
c.CAD d. Atherosclerosis
Ans : a

106. Symptom associated with polycythemia in children is ……….


a. Cardiomegaly b. Polyuria
c.Edema d. Flushed face
Ans : d

107. A dull sound heard during chest percussion, it is due to the presence of
a. Mass b. Fluid
c.Bone d. Air
Ans : a

108. Best method to check fluid replacement in adult is


a. Urine output b. Blood pressure
c.Respiration d. Pulse pressure
Ans : a

109. Body part goes midline to away is termed as


a. Abduction b. Adduction
c.Supination d. Flexion
Ans : a

110. While recording oxygen with pulse oximetry of a newly married patient, the nurse
should do
a. Remove nail polish b. Remove bangles
c.Used toe finger d. None of the above
Ans : a

111. Best intervention for hemorrhagic shock is to provide total


a. 1-2 liter fluid b. 2-3 liter fluid
c.3-4 liter fluid d. 4-5 liter fluid
Ans : c

112. To check the skin turgor nurse should do


a. Pinch the skin gently and observe
b. Measure the skin fold thickness
c. Ask the client to take 2-3 L of fluids
d. Ask the client to lay down in prone position
Ans : a

113. A neonate doesn’t pass meconium after 24 hours of delivery and neonate
suspected for Hirschsprung’s disease. The next action should be
a. Prepare for surgery
b. Insert feeding tube
c. Insert rectal tube and give enema of normal saline
d. Provide Ryle’s tube feeding
Ans : a

114. A patient doesn’t pass urine and his kidney works normal. The term used to
describe this condition is
a. Retention b. Renal failure
c.Oliguria d. Anuria
Ans : a

115. Least effective therapy in OCD is


a. Exposure b. Cognitive therapy
c.Medication d. Physical exercise
Ans : d

116. A client sleep between conversion, describe as


a. Sleep apnea b. Narocolepsy
c.Somnambulism d. Insomnia
Ans : b

117. A patient shows which of the following symptom of overdose of lithium


a. Polyuria and drinking water
b. Weight loss
c. Leukocytopenia
d. Hyporeflexia
Ans : a

118. “ Scheduled activity “ therapy used for the patient with


a. Negative symptoms of schizophrenia
b. Depression
c. Mania
d. OCD
Ans : b

119. Body part goes midline to away is termed as


a. Abduction b. Adduction
c.Supination d. Flexion
Ans : a
120. All are the risk factors of cardiac disease, EXCEPT
a. Office job b. Age above 70 years
c.Increased HDL level d. Increased LDL level
Ans : c

121. Person’s lab value shows cholesterol level 300 mg. the nurse advised to the person
a. To check lipid profile b. Modify the dietary pattern
c.Collect history regarding ADL d. Reduce the weight
Ans : a

122. Hormone responsible for milk ejection is


a. Oxytocin b. Prolactin
c.ADH d. Insulin
Ans : a

123. Which position is shown in picture


picture

a. LOP b. LOA
c.ROP d. ROA
Ans : d

124. An OCD patient becomes good and family stops the treatment before 3 months.
Nurse teaches the family that ………………….
a. Treatment is for long time
b. No dependence occur with the treatment
c. After total cure doctor stop the treatment
d. No more treatment is needed for the client
Ans : a

125. A person thinks that somebody follows him because he had secret paper in his
briefcase. It is
a. Delusion of persecution b. Idea of reference
c.Idea of suspect d. Delusion of grandeur
Ans : a

126. This is not the effect of cold therapy


a. vasodilation b. Vasoconstriction
c. Reduce pain d. Stop bleeding
Ans :a
127. Pain scale used for the assessment of pain in postoperative patients is
a. VAS b. VAP
c.AVP d. VSP
Ans :a

128. Shakir tape is used to measure


a. Mid upper arm circumference
b. Head circumference
c. Chest circumference
d. Total length
Ans : a

129. While doing ET suction, suction catheter should insert


a. Till obstruct the catheter in trachea
b. Total length of catheter
c. Tip of the nose to ear and ear to chest
d. Insert the catheter beyond carina
Ans : a

130. Normal anion gap is


a. 1-4 mEq/L b.4-8 mEq/L
c.18-24 mEq/L d. 8-16mEq/L
Ans : d

131. After inspection which technique should be used for proper abdominal
assessment
a. Auscultation b. Palpation
c.Observation d. Percussion
Ans : a

132. In the third stage of labor uterus become hard. The immediate action of nurse is
a. Uterine massage b. Increase oral fluids
c.Record temperature d. shift to ward
Ans : a

133. Which is used first to treat hypovolemic shock


a. Plasma b. Fresh blood
c.Whole blood d. Packed cells
Ans : c
134. A 38 year old day wants to use spacing method. She is smoking one packet of
cigarettes daily. Which method of contraception is contraindicated in this women?
a. Oral contraceptive pills b. Diaphragm
c.Copper T d. Female condom
Ans : a

135. After intercourse how long female diaphragm should kept in the genital tract?
a. 6 hours b. 1 hour
c.24 hour d. 12 hour
Ans : a

136. Agent , host, environment are


a. Risk factors b. Infection
c.Components of model d. Clinical signs
Ans : c

137. Look at the picture and identify


Picture
a. Phototherapy b. Radiant warmer
c.Heater d. X-ray
Ans : a

138. In the following, which is not a sign of depression


a. Anhedonia b. Irritability
c.Restless d. Hopeful
Ans : d

139. First case found is known as


a. Index case b. Primary case
c.Secondary case d. Tertiary case
Ans : a

140. The cause of gastric cancer is


a. H. pylorai b. Acidic nature
c.Less peristalsis d. Activity daily living
Ans : a

141. Method used for temperature management in radiant warmer is


a. Control mode b. Manual mode
c.Servo mode d. None of them
Ans: c

142. Insulin secretes by


a. Beta cells of pancreas b. Alpha-cells of pancreas
c.Gamma cells of pancreas d. Gallbladder
Ans : a

143. Increase in number of cells are called


a. Hyperplasia b. Hypoplasia
c.Metaplasia d. Dysplasia
Ans : a

144. After birth heart rate of neonate is 100/minute. The immediate next action of
nurse is
a. Rub the back of neonate & tap stimulation at the sole
b. Provide oxygenation
c. Start cardiopulmonary resuscitation
d. Handover the baby to mother
Ans : a

145. Haemothorax occurs in


a. Pleural space b. Thoracic cavity
c.Peritoneal space d. Periosteum
Ans : a

146. Method used to transport the vaccine from one place to another is
a. Cold chain b. Vaccine carrier
c.Deep fridge d. Ice lined refrigerator
Ans : a

147. Which symptom is not shown in the patient with anxiety


a. Hallucination b. Nervous behavior
c.Worry d. Restlessness
Ans : a

148. Nursing action to reduce physiological jaundice in a neonate is


a. Give IV fluids
b. Start single surface phototherapy
c. Start double surface phototherapy
d. Place the child in billi-blanket
Ans : a

149. Intractable pain is


a. Intermittent pain b. Intermediate pain
c.Constant pain d. Wide spreading pain
Ans : c

150. See the picture and record PR interval


Picture
a. 0.16 b. 0.20
c.0.24 d. 0.8
Ans : c

151. An alcoholic patient stop the alcohol consumption before 24 hours. He faced
tremors, agitation and confusion. What is the diagnosis
a. Alcohol dependence syndrome with alcohol consuming persons
b. Normal finding in alcohol consuming persons
c. Wernicke’s encephalopathy
d. Psychosis
Ans : a

152. Which is not finding in dengue patient


a. Haemodilution b. Thrombocytopenia
c.Increased capillary permeability d. Hypertension
Ans : d

153. A patient gets infection from indwelling catheter. It is an example of


a.iatrogenic infection b. Nosocomial infection
c.Droplet infection d. Contamination
Ans : a

154. A diabetic patient put on NBM (NPO) for CT scan. What the nurse do
a. Postponed the insulin dose
b. Administer the hypoglycemic agents as usual
c. Tell the patient it is not possible to be on NPO status
d. Give IVF
Ans : a

155. Before giving RT feed to a child nurse check the placement of tube
a. Every time before feed b. one time in a day
c.After feed d. 3 times in a day
Ans : a

156. Position giving to mother for caesarian section


a. Supine position with pillow under hip
b. Semi fowler position
c. Prone position
d. Lateral position
Ans : a

157. Impairment in consciousness and cognition for short time is the


a. Delirium b. Dementia
c.LOC d. Parkinson
Ans : a

158. Which cell acts as a phagocytic


a. Monocyte b. Thrombocyte
c.Macrophage d. RBC
Ans : c

159. After giving epidural block in pregnant mother, nurse should check for
a. Hypotension b. Confusion
c.Urine output d. Tachypnea
Ans : c

160. Process of suction of the baby


a. First mouth than nose b. Nose than mouth
c.Only nose d. Only mouth
Ans : a

161. A farmer was spraying some liquid in the farm, suddenly he falls down and
brought by his relatives in emergency room with low BP, constricted pupil and
unconscious state. You will know that these are the symptom of which type of
poisoning
a. Lead poisoning b. Arsenic poisoning
c.Phenol poisoning d. Latex poisoning
Ans : b
162. A nurse has given injection ondensetron to a patient. After 10 minutes, fingers of
patient become blue in color, this is because of………………..
a. Intra arterial injection b. Infiltration
c.Phlebitis d. Thrombus formation
Ans : a

163. Surgery of pyloric stenosis


a. Pyloromyotomy b. Nissen fundoplication
c.Sweanson procedure d. Duhamel procedure
Ans : d

164. Coomb’s acts as


a. Direct Coomb’s test is used to test autoimmune hemolytic anemia
b. Indirect coomb’s test detects antibodies against RBCs that are present
unbound in the patient’s serum.
c. Coomb’s tests are performed using RBCs or serum
d. All the above
Ans : d

165. Disulfiram acts as


a. Antabuse b. Antidote
c.Analgesics d.Antipyretics
Ans : a

166. Extra pyramidal symptom


a. Dystonia b. Akathisia
C. parkinsonism d. All the above
Ans : d

167. Pheochromocytoma is…………….


a. Tumor originating in chromaffin cell
b. Tumor of kidney
c. Tumor of pancreas
d. Tumor of osteocyte
Ans : a

PGIMER 2016
1. A characteristic of infants and young children who have experienced maternal
deprivation is
a. Tendency toward overeating b. Responsiveness to stimuli
c.Proneness to illness d. extreme activity
Ans : c

2. Which of the following is an appropriate nursing action when caring a patient


who has a radium implant for cancer of cervix?
a. Restrict the visitors to a 10- minute stay
b. Store urine in a lead – lined container
c. Wear a lead apron when giving care
d. Avoid giving IM injection to gluteal region
Ans : a

3. Caput succedaneum is:


a. Birth mark
b. Swelling on the face of the baby
c. Collection of fluid under scalp
d. Swelling will not cross the suture line
Ans : c

4. When a patient is in liver failure, which of the following behavioral changes is


the most important assessment to report?
a. Shortness of breath b. Lethargy
c.Fatigue d. Nausea
Ans : d

5. Under integrated management of neonatal and childhood illness care is


rendered to:
b. Only newborn
c. Under five children
d. 2 months to 5- year old children
e. Age up to 2 month and 2 mother to 5 year old children
Ans : b

6. Which of the following clinical finding indicates the patient is experiencing


hypokalemia?
a. Edema b. Muscle spasms
c.Kussmaul breathing d. Abdominal distention
Ans : b
7. A 20- month – old is to receive isolyte –p 240 ml IV in 6 hours. The drop factor
of the mini dropper is 60 drops/ml. the nurse should regulate the IV to run at:
a. 36 drops per minute b. 40 drops per minute
c.32 drops per minute d. 20 drops per minute
Ans : b

8. Total blood volume of a newborn at birth is:


a. 80 mL/kg b. 100 mL/kg
c. 125mL/kg d. 150 mL/kg
Ans : a

9. The enzyme released by the sperm which allows penetration of the ovum is
called
a. Amylase b. Hyaluronidase
c.Lipase d. Trypsin
Ans : b

10. Proteins are required for


a. Weight reduction b. absorption of calcium
c.Maintenance of body tissues d. Synthesis of steroid hormones
Ans : c

11. A patient’s physician orders unclear cardiography and makes an appointment


for a thallium scan. The purpose of injecting a radioisotope into blood stream is
to detect
a. Normal versus abnormal tissue
b. Damage in areas of the heart
c. Ventricular function
d. Myocardial scarring and perfusion
Ans : d

12. The reason for using the parenteral route to administer medication includes:
a. Parenteral medication last longer than an oral medication
b. It is the least expensive method
c. It is easier to measure an accurate dose
d. The parenteral route allows more rapid absorption than the oral route
Ans : d

13. A skin infection that can be a sequel of a staphylococcal infection is:


a. Herpes simplex b. Scabies
c.Vertigo d. Impetigo
Ans : d

14. The mean cardiac output of the fetus is:


a. 100 mL/kg/minute b. 225 mL/kg/minute
c. 350 mL/kg/minute d. 415 mL/kg/minute
Ans : d

15. Which of the following is a preparation of choice for a patient who has been
admitted in ED with an open contaminated injury and no recent history of
tetanus immunization?
a. DTP vaccine b. Tetanus toxoid
c.Tetanus antitoxin d. Tetanus immunoglobulin
Ans : d

16. Normally newborns lose weight in first week of life


a. 1%of birth weight b. 5% of birth weight
c. 10%of birth weight d. 20% of birth weight
Ans : c

17. The immunity developed as a result of infection by pathogenic organism is


a. Active immunity b. Passive immunity
c.Natural immunity d. Acquired immunity
Ans : d

18. Anti D should be given following Rh+ve delivery within:


a. 06 hours b. 24 hours
c.72 hours d. 07 days
Ans : c

19. ART-DOTS linkages are being established at all…………………


a. Health centers centers
b. ART centers under AIDS control program
c. Hospitals
d. TB centers
Ans : b

20. The population served by a sub center in plain areas is


a. 1000 b. 5000
c.30000 d. 80000
Ans : b

21. The vaccine used for the prevention of cervical cancer is


a. BCG b. IgG
c.Hep B d. HPV
Ans : d

22. The first person becoming sick in an epidemic is called


a. Primary case b. Secondary case
c.Contact case d. Index case
Ans : a

23. Most appropriate age for early primary immunization against TB, Diphtheria,
whooping cough, tetanus,polio,measles and hepatitis B is
a. Birth to 6 months b. Birth to 9 months
c.Birth to 1 year d. Birth to 18 months
Ans : b

24. EPI includes all vaccines except


a.DPT b. MMR
c.Polio d. BCG
Ans : b

25. A 28 –year-old male patient is admitted to the hospital for a suspected brain
tumor. While assessing this patient ,the nurse would keep in mind that the
most reliable index of cerebral status is
a. Pupil response b. Deep tendon reflexes
c.Muscle strength d. Level of consciousness
Ans : d

26. Pain felt in calf muscle when the foot is dorsiflexed with the leg extended at
the knee is called is:
a. Kerning’s sign b. Spalding’s sign
c.Homan sign d. Goodell’s sign
Ans : a
27. A young patient who was hit by a car was fortunate because the level of the
injury did not interrupt his respiratory function. The cord segments involved
with maintaining respiratory function are
a. Thoracic level 5 and 6 b. Thoracic level 2 and 3
c.Cervical level 7 and 8 d. Cervical level 3 and 4
Ans : d

28. Physiological anemia in pregnant women is a result of


a. Poor dietary intake of iron
b. Increases erythropoiesis
c. Increased blood volume of women
d. Increased detoxification demands
Ans : c

29. Strawberry spot vagina is seen in


a. Candida b. Trichomonas
c.CMV d. Herpes simplex
Ans : b

30. A way to promote trust with a patient is to:


a. Allow family members to visit whenever they want
b. Assure the patient that her doctor is available always
c. Follow through when you say you will do something
d. Talk with her at length,about her life ,like and dislikes
Ans : c

31. The first dose of vit A is given at the age of


a. 1 ½ year b. 2 year
C.9 months d. 6 months
Ans : c

32. Normally by one year of age the birth weight of an infant should increase by
a. 1.5 times b. 2 times
c.3 times d. 4 times
Ans : b

33. Tertiary prevention is needed in with stage of the natural history of disease?
a. Pre-pathogenesis b. Pathogenesis
c.Predromal d. Terminal
Ans : d
34. A Non –stress test is prescribed for a pregnant and nurse about the procedure.
The nurse tells the client that:
a. The test is an invasive procedure and requires that an informed consent be
signed
b. The test will take about 2 hours after procedures is completed
c. An ultrasound transducer that records abdomen where the fetal heart
sound is heard most clearly
d. The fetus is challenged or stressed by necessary information
Ans : c

35. The physician has ordered a 24 hours urine specimen. After explaining the
procedure to the patient, the nurse collects the first specimen. This specimen is
then
a. Discarded, then the collection begins
b. Saved as part of the 24 hours collection
c. Tested , then discarded
d. Placed in a separate container and later added to the collection
Ans : a

36. Communicability of measles declines


a. After onset of fever b. During prodromal period
c.At the time of eruption d. After appearance of rash
Ans : d

37. The nurse will evaluate for the most significant complication in patients
undergone chronic peritoneal dialysis, which is
a. Pulmonary embolism b. Hypotension
c.Dyspnea d. Peritonitis
Ans : d

38. BCG, Measles, MMR vaccines should be stored in


a.100 to 1200C b. 200 to 2500C
c.20 to 80C d. 150 to 2000C
Ans : c

39. When a patient’s medical record is needed as evidence for a legal action, you
are aware that the record is property of
a. The patient b. The patient’s lawyer
c.The court d. The health are agency
Ans : d

40. In India which of the following function as first Referral units ( FRUs) ?
a. Sub centers b. Primary health centers
c.Community health centers d. Regional hospitals
Ans : c

41. Which of the clients would nurse identify as being at most risk for developing
disseminated intravascular coagulation (DIC)?
a. A 4TH gravida delivered 8 hours age and has lost 500 ml of blood
b. A 2nd gravid diagnosed with dead fetus syndrome
c. A primigravida with mild pre-ecclampsia
d. A primigravida who delivered a 3.5 kg baby 3 hours ago
Ans : b

42. The principle of at risk approach is


a. Something for all b. More for the needy
c.All for some d. All for all
Ans : b

43. A patient has the diagnosis of left ventricular failure and a high pulmonary
capillary wedge pressure ( PCWP). The physician orders dopamine to improve
ventricular function. The nurse will know the medication is working if the
patient’s
a. Blood pressure rises b. Blood pressure decreases
c.Cardiac index fails d. PCWP rises
Ans : a

44. The main objective of intensive IEC actives for leprosy program is :
a. Cure of illness b. Reduction of stigma
c.Case identification d. Eradication of disease
Ans : d

45. Promotion of infant and young child feeding practice is encompassed along
with continuation of complementary feeding up to :
a. One year of life b. Two years of life
c.Three years of life d. Four years life
Ans : b
46. Thrombophlebitis is a common complication following vascular surgery.
Which of the following signs indicates that a possible thrombus has occurred ?
a. Kernig’s sign b. Hegar’s sign
c.Homan’s sign c.Brudzinski’s sign
Ans : c

47. During a retention catheter insertion or bladder irrigation ,the nurse must use
a. Sterile equipment and wear sterile gloves
b. Clean equipment and maintain medical asepsis
c. Sterile equipment and maintain medical asepsis
d. Clean equipment and technique
Ans : c

48. Saffron colored meconium seen in:


a. Post maturity b. TB
c.Breech d. Normal in appearance
Ans : a

49. The extra calories needed by a woman having singleton, pregnancy


a. 100 Kcal b. 200Kcl
c.300 Kcl d. 500 Kcl
Ans : c

50. Which of the following principle guides nurse’s priorities at a disaster caused
by a collapsed building in an earthquake?
a. Hemorrhage necessitates immediate care to save most lives
b. Those requiring minimal care are treated first so that they can help others
c. Those with head injuries are treated first so that their care more complex
d. Children should get highest priority because they have the greatest life
expectancy
Ans : b

51. Mantoux test is done to diagnose


a. Malaria b. Filiaria
c.Tuberculosis d. Typhoid
Ans : c

52. Bitot’s spots are due to the deficiency of


a. Vit B b. Vit A
c.Vit D d. Carbohydrates
Ans : b

53. A 45 years old patient Mrs. Bhagwanti devi has been admitted to the hospital
for an abdominal hysterectomy following a diagnosis of uterine cancer. Result
of lab tests indicate that the patient’s WBC is 9800/cu mm. the patient’s WBC
is 9800/cu mm. The most appropriate intervention is to
a. Call the operating room and cancel the surgery
b. Notify the surgeon immediately
c. Take no action as this is a normal value
d. Call the lab and have the test repeated
Ans : c

54. The results of mantoux test are read after


a. Six hours b. 24 hours
c.48 hours d. 72 hours
Ans : c

55. Chikungunya is a non-fatal …………………… illness:


a. Viral b. Bacterial
c.Protozoan d. Plasmodium
Ans : a

56. Following a liver biopsy, the highest priority assessment of the patient’s
condition is to check for
a. Pulmonary edema b. Uneven respiratory pattern
c.Hemorrhage d. Pain
Ans : c

57. Strongest stimulus of lactation is by:


a. Metoclopramide b. PPH
c.Bromocriptine d. Sucking
Ans : d

58. The number of deaths per 1,000 population per year in a given community is
known as
a. Crude death rate b. Infant death rate
c.Neonatal death rate d. Perinatal death rate
Ans : a
59. A pregnant woman admitted in labor room is attached to external electronic
fetal monitor; nurse should do initial nursing assessment which of the
following?
a. Identifying types of accelerations
b. Assessing baseline fetal heart rate contractions
c. Determining intensity of the contractions
d. Determining intensity of the contractions
Ans : b

60. When serum bilirubin rises on 3rd and 4th day after birth of the baby due to
inefficient hepatic function is known as:
a. Pathological jaundice b. Physiological jaundice
c.Obstructed jaundice d. Viral hepatitis
Ans : b

61. A female patient Mrs.Seema Gupta has a cesium needle implanted in her cervix.
She asks the nurse if she may got out of bed to go to the bathroom. The
appropriate response is to tell her:
a. She may not get out of bed while the needle is implanted
b. She may get out of bed with the nurse’s help
c. The nurse will have to get a physician’s order for her to get out of bed
d. She must stay in bed, but she can move around to be more comfortable
while the needle is implanted.
Ans : d

62. Rice water stool is the typical sign of


a. Cholera b. Hepatitis A
c.Hepatitis B d. Dysentery
Ans : a

63. The assessment finding that should be reported immediately in the patients is
a. Nausea and vomiting b. Abdominal pain
c.Decreased bowel sounds d. Shortness of breath
Ans : b
64. A nurse is assessing a child for dehydration. The nurse determines that the
child is moderately dehydrated if which of the symptom is noted:
a. Flat fontanels b. Moist mucous membrane
c.Pale skin color d. Oliguria
Ans : d
65. The nurse knows that the most informative measurement for determining
cardiogenic shock is
a. Arterial blood pressure b. central venous pressure
c.Pulmonary artery pressure d. Cardiac index
Ans : a

66. Which of the following is the appropriate nursing intervention for a patient
with a terminal illness who is passing through the acceptance stage?
a. Allowing the patient to cry
b. Encouraging unrestricted visiting
c. Explaining the patient what is being done
d. Being around though not speaking
Ans : d

67. A female patient has a laparoscopic cholecystectomy this morning. She is now
complaining of right shoulder pain. The nurse would explain to the patient this
symptoms is
a. Common following this operation
b. Expected after general anesthesia
c. Unusual and will be reported to the surgeon
d. Indicative of a need to use the incentive spirometer
Ans : a

68. Which immunization produces a permanent scar?


a. DPT b. BCG
c.Measles vaccination d. Hepatitis B vaccination
Ans : b

69. To disinfect 20 liters of water, one should use a single chlorine tablet of
a. 0.5 g b. 1.5 g
c.2.5 g d. 3.5 g
Ans : a

70. A patient was in an automobile accident, and sustained a head injury, following
admission to the hospital, a diagnosis of increasing intracranial pressure was
made. The nursing intervention appropriate in the care of this patient is to
a. Tech controlled coughing and deep breathing
b. Provide a quiet and brightly lit environment
c. Elevating the heard 15 to 30 degree
d. Encourage the intake of clear fluid
Ans : d

71. Expenditure in health sector on public health is about ……………of the GDP in
2015-16
a. 1% b. 2%
c.5% d. 6%
Ans : a

72. Which of the following system of medicine is of Indian origin?


a. Homoeopathy b. Siddha
c.Unani – tibb c. Acupuncture
Ans : b

73. Normal length of newborn at birth is:


a. 20-25 cms b. 45-50 cms
c.72-80 cms d. 78-85 cms
Ans : b

74. Immunity by antibody formation during the course of a disease is :


a. Active natural immunity b. Active artificial immunity
c.Passive natural immunity d. Passive artificial immunity
Ans : a

75. Dyspnea associated with congestive heart failure is primarily due to


a. Blockage of a pulmonary artery by an embolus
b. Accumulation of fluid in the interstitial spaces and alveoli of the lungs
c. Blockage of bronchi by mucous secretions
d. Compression of lungs by the dilated heart
Ans : b

AIIMS Bhopal 2016


1. Mozilla firefox is a(n):
a. Utility program b. Anti-plagiarizing program
c.Internet browser d. Operating system
Ans : c
2. Find the opposite of the key word.
Camouflage:
a. Acknowledge b. Ambush
c.Disguise d. Divulge
Ans : d

3. The most widely use OS for mobile phones developed by Google is :


a. Ubuntu b. Magellan
c.Amoeba d. Android
Ans : d

4. Find the meaning of the underlined idiom.


I know all the “ins and outs” of this job
a. Importance b. Usefulness
c.Difficulties d. Details
Ans : d

5. Find one word for the given group of words Lion and tiger,
a. Carnivore b. Insectivore
c.Herbivore d. Ominvore
Ans : a

6. Who is elected as the new chief minister of Gujarath in August,2016


a. Amit Shah b. Anandiben Patel
c.Nitin Patel d. Vijay Rupani
Ans : d

7. Find the word which best expresses the meaning of the key word. Manoeuvre :
a. Manipulate b. Move
c.Exercise d. Act
Ans : b

8. Dengue fever is caused by bite of which mosquitoes?


a. Culex b. Anopheles
c.Man Sonia d. Aedes
Ans : c

9. ALU in the computer system stands for :


a. Application logic unit b. Array logic unit
c.Arithmetic and logic unit d. Alternative logic unit
Ans : c

10. Who created history by winning record- breaking 23rd gold medal of Olympics history
at Rio Olympics, 2016?
a. Carl Lewis b. Mark spitz
c.Usain Bolt d. Michael Phelps
Ans : d

11. Chemical coagulant used to treat raw river water is :


a. Alum b. Sodium chloride
c.Cholrine d. Potassium permanganate
Ans : a

12. MS-DOS is an Operating system that has:


a. Command line interface b. Option for Mobile devices
c.Graphical user interface d. Open source origin
Ans : a

13. Which of the following is the correct sequence of the smallest to the largest unit of
storage size ( read from left to right)?
a. Mega-giga-kilo-peta b. Kilo-mega-giga-tera-peta
c.Kilo-mega-giga-peta-tera d. Kilo-mega-peta-tera-gign
Ans : b

14. The normal composition of oxygen in the environment is:


a. 40% b. 30.93%
C.20.93% d. 50%
Ans : c

15. If clock of Delhi displays 1:00 PM then at same time clock of Dubai will display
………………
a. 2:00PM b. 12:00PM
c.11:30 AM d. 10:30AM
Ans : c

16. How many stores will be opened up during 2016-17 under Prime Minister’s Jan
Aushadhi Yojana as per union budget for the year 2016-17?
a. 3000 b. 2000
c.4000 d. 1000
Ans: a
17. In the following question find the word which means the same as the given group of
words. “Period of one thousand years”
a. Decade b. Century
c.Centenary d. Millennium
Ans : d

18. Which of the following is not a characteristic of safe and wholesome water ?
a. Free from chemicals b. Free from pathogens
c.Pleasant to taste d. Rich in iron content
Ans : d

19. Indira awaas Yojna is meant for which population?


a. Semi – urban b. Rural
c.Urban and Rural d. Urban
Ans : b

20. “Dumhal” Dance is performed in the …………….state of India


a. Jammu & Kashmir b. Punjab
c.Assam d. Gujarat
Ans : a

21. Which of the following is NOT a characteristic of a social group?


a. We – feeling b. Similar ethnic background
c.Reciprocal relationship d. Common interest
Ans : c

22. The umbilical cord contains:


a. Two arteries and one vein b. One arteries and one vein
c.Two arteries and two veins d. One artery and two veins
Ans : a

23. ‘Man is a social animal’,-Who said this?


a. Cristo b. Spencer
c.Pluto d. Aristotle
Ans : d

24. Enuresis means:


a. Loss of the control of urine b. Breast holding
c.Loss of control of stool d. Loss of control of vomit
Ans : a

25. Principle of effective leadership is least likely to involve:


a. One group commanded by one leader
b. One group should always have an objective
c. Accountability is delegated to the members of term
d. Patient first policy is applied
Ans : c

26. Which of the following are inventory control methods?


a. HDL analysis, FED analysis
b. DRF analysis , DEL analysis
c. ABC analysis, VED analysis
d. FED analysis, LDL analysis
Ans : c

27. The ability of human body to protect itself from pathogens is known as :
a. Secretions of hormones b. Hypersensitivity
c.Locomotion d. Immunity
Ans : d

28. Which of the following is fourth cranial nerve?


a. Abducens b. Trochlear
c.Trigeminal d. Facial
Ans : b

29. The major portion of calcium is found in which part of body?


a. Blood b. Heart
c.Muscle d. Bone
Ans : d

30. The first aid treatment for choking is :


a. Call for ambulance
b. Give something to drink
c. Try to remove the object with your fingers
d. Give five sharp back blows between shoulder blades
Ans : d
31. The power which is exercised because of appointment to a higher position is referred as
a. Exclusive power b. Expert power
c.Reward power d. Referent power
Ans : a

32. The nurse is assessing a patient’s abdomen –which examination technique should the
nurse use first?
a. Inspection b. Auscultation
c.Percussion d. Palpation
Ans : a

33. How many ANC visits on a minimum should be recommended for an ANC mother?
a. 6 visits b. 1 visits
c.4 visits d. 5 visits
Ans : c

34. A group of people organized for particular purpose is known as?


a. Community b. Institution
c.Society d. Association
Ans : d

35. What name did freud give to his model of development which comprise id. Ego and
superego?
a. Genetic model b. Unconscious model
c.Topographical model d. Structural model
Ans : d

36. Which of the following is the element of difference between community and society?
a. Definitive locality b. A group of persons
c.Sentiment of oneness d. Likeness of interest
Ans : a

37. If a person has an object implanted in his hand, the first aider must:
a. Only break the object if it hinders the transportation
b. Break the object form where it is visible
c. Immobilize the object using sterile pad
d. Try to remove the object pulling it
Ans : c

38. What does a society exclude?


a. Time boundless b. Differences
c.Reciprocity d. Interdependence
Ans : a
39. Which committee was set up to see the general condition of nurse?
a. High power committee b. Bhore committee
c.Mudaliar committee d. Mukherjee committee
Ans : a

40. What is the approx. total population of India as per provisional data. 2011 census?
a. 121 crore b. 150 crore
c.160 crore d. 180 crore
Ans : a

41. Which of the following is the purpose of hand washing ?


a. To provide comfort b. To prevent cross infection
b. To have aesthetic feeling d. To promote circulation in the hand
Ans : b

42. Anxiety is caused by:


a. Hostility turned to self b. Masked depression
c.An objective threat d. A subjective, perceived thereat
Ans : d

43. What is hyperemesis gravidraum ?


a. Food craving b. Excessive vomiting
c.Morning sickness d. Lethargy
Ans : b

44. Immediate first aid treatment for dog bite is:


a. Suture the big wound
b. Cover with dressing
c. Wash with soap and water for 15 minutes
d. Apply antiseptic lotion
Ans : c

45. Dissemination of research can be done by following methods. Except:


a. Oral presentation b. Publication in scientific journal
c.Poster presentation d. Information discussion
Ans : d

46. The ratio of cardiac compression and artificial breathing is :


a. 60:6 b. 15 : 2
c.30 :2 d. 12:3
Ans :c

47. The color used for tagging a dead body is :


a. Yellow b. Black
c.Red d. Green
Ans : b

48. The definition of health by WHO includes all, Except:


a. Mental b. Physical
c.Reproductive d. Social
Ans : c

49. The neural tube defect among newborn can be prevented by providing:
a. Folic acid to the mother in first trimester
b. Vitamin K to the mother in first trimester
c. Iron and folic acid throughout pregnancy
d. Iron to the mother in 3rd trimester
Ans : a

50. Which of the health education agency is at national level?


a. Press Information Bureau
b. Central Health education bureau (CHEB)
c. Doordarshan
d. IEC Bureau
Ans : b

51. An anganwadi worker caters to what size of population?


a. One thousand b. One million
c.One hundred c. One lakh
Ans : a

52. Which is NOT the responsibility of a head nurse/Ward –in charge?


a. Preparing patients bill b. Delegation
c.Nurses assignment d. Patient care
Ans : a

53. Secondary level of prevention of disease includes:


a. Early diagnosis and treatment
b. Rehabilitation and disability limitation
c. Specific protection and screening
d. Screening and immunization
Ans : a

54. What is the normal blood cholesterol level?


a. 500-100 mg/dl b. More than 200 mg/dl
c.Less than 200 mg/dl d. 1000-12 mg/dl
Ans : c

55. Attributes of good leader involves which of the following


a. Aware of member’s strengths and weaknesses
b. Suppresses the voice of dissent
c. Don’t hold accountability for wrong doings
d. Not sure about his/her skills
Ans : a

56. Hallucination is ……………………….


a. Misinterpretation of real external stimuli
b. Perception without stimuli
c. Clear perception of stimuli
d. Wrong perception of stimuli
Ans : b

57. A muscular pouch – like sac that lies slightly to the left of abdomen to store food
temporarily is known as :
a. Stomach b. Urinary bladder
c.Gallbladder d. Lungs
Ans : a

58. What are the professional ethics?


a. Code of professional ethics are not often established to guide
b. Professional accepted standards are not present
c. Standards of personal & business behavior, values,& guiding principles
d. Not performing their job functions
Ans : c

59. Nursing profession can only be practiced after


a. Approval b. Valuation
c.Licensure d. Accreditation
Ans : c
60. Which one of the following helps to reduced anxiety and postoperative pain?
a. Rest and sleep b. Psychological counseling
c.Per-operative teaching d. Pre-operative checklist
Ans : c

61. How many calories provided by 1 gram of protein?


a. 8 b. 10
c.4 d. 6
Ans : c

62. Exogamy is defined as :


a. Compassionate marriages b. Marriage outside a group
c.Experimental marriages d. Marriages within the group
Ans : b

63. What are NOT the misconception related to mental health?


a. It is possible to prevent mental health illness
b. Mental problems do not affect health
c. People with mental health problems are violent and unpredictable
d. Children do not experience mental health problems
Ans : a

64. With respect to the daily calorie requirement for a normal being the contribution via
carbohydrate should be :
a. 40-50% of total requirement
b. 50-70% of total daily requirement
c. 80-90% of total daily requirement
d. 90-100% of total requirement
Ans : b

65. Displacement, substitution and projection and examples of which of the following ?
a. Psychosocial stage development
b. Defense mechanism
c. Psychosexual stage of development
d. Neurotic need
Ans : b

66. Which is a unit of care in practice of community health nursing?


a. Individual b. Family
c.Community d. Significant others
Ans : b

67. Which of the following is NOT a method of patient assignment?


a. Functional method b. Team nursing
c.Patient assignment d. Duty roster methods
Ans : d

68. Which of the following are fat soluble vitamins?


a. A,B,C,D b. B,E,C,K
c.A,D,B,C d. A,D,K,E
Ans : d

69. Which of the following is NOT an important element in physical environment


surrounding a job that can affect motivation?
a. Plant and animal in the environment
b. Lighting
c. Noise
d. Temperature
Ans : a

70. Which of the following is measure of central tendency?


a. Mean b. Range
c.Standard deviation d. Ratio
Ans : a

71. Normal duration for involution of uterus is :


a. 6 weeks b. 2 weeks
c.1 month d. 2 months
Ans : a

72. Validity of a tool refers to:


a. Feasibility b. Consistency
c.Accuracy d. Acceptability
Ans : c

73. Cancer of cervix is caused by:


a. Human immunodeficiency virus b. Human diploid virus
c.Human papilloma virus d. Rota virus
Ans : c
74. Culture has importance for the group because
a. It provides stability to the group
b. It satisfies human needs of food and shelter
c. It makes off one group from another
d. It keeps social relationship intact
Ans : c

75. Which of the following gland secretes hormone to maintain normal metabolic rate of
the body?
a. Testes b. Pancreas
c.Adrenal d. Thyroid gland
Ans : d

76. Which of the following statement is not true?


a. Family is system of relationship between parents and children
b. Family is a group of person who are related to each other by religion
c. Family is a biological unit
d. Family is a group defined by social relationship
Ans : b

77. Total number of deciduous teeth in children are:


a. 16 b. 14
c.12 d. 20
Ans : d

78. Norms in sociology indicates:


a. Ethical ideas of society
b. Tendency of human being to move up
c. Normal standard of social behavior
d. Standardized generalization about normally expected modes of behavior
Ans : d

79. Communication process includes


a. Message, messenger, channel
b. Communicator, environment feedback
c. Communicator, message, channel, receiver, feedback
d. Communicator, receiver, feedback
Ans : c
80. What is the dose of vitamin A for an infant at nine months of age?
a. 4 lakh IU b. 3 lakh IU
c.2 lakh IU d. 1 lakh IU
Ans : d

81. Sample in research is referred as:


a. Representative of population b. High risk study subjects
c.High risk population d. Population living in a defined area
Ans : a

82. Telling the truth is a practice of which ethical principle?


a. Non – maleficence b. Justice
c.Beneficence d. Veracity
Ans : d

83. The tools used for data collection are:


a. Experimental, non-experimental
b. Participant observation, information booklet
c. Survey, observation check list
d. Questionnaire, interview schedule, observation checklist
Ans : d

84. What immediate nursing intervention will you take in case of blood transfusion
reaction?
a. Monitor vital signs
b. Inform blood bank
c. Inform the physician
d. Stop the transfusion immediately and monitor vital signs
Ans : d

85. The pacemaker of heart is :


a. Purkinje fibers b. Bundle of his
c.SA node d. AV node
Ans : c

86. Absence of breathing is known as:


a. Eupnea b. Apnea
c.Tachypnea d. Bradypnea
Ans : b
87. What are the signs of shock?
a. Lethargy and slow pulse rate
b. Cold clammy skin, unconsciousness and rapid pulse rate
c. Rapid pulse rate and unconsciousness
d. Sweating and forgetfulness
Ans : b

88. Which of the following is a DNA virus?


a. Hepatitis B b. Hepatitis C
c.Hepatitis A d. Hepatitis D
Ans : a

89. Which position is to be provided to the patient while performing mouth care to an
unconscious patient?
a. Fowler’s position b. Side lying
c.Knee chest d. Supine
Ans : b

90. What are the trends in psychiatric nursing?


a. Practice of nursing in new and emerging role
b. Dilemmas in care
c. Custodial care
d. Certificate program for graduate and experienced nurses
Ans : c

91. Which of the following is a significant component of pain assessment?


a. Intensity b. Cause
c.Causing factor d. Effect
Ans : a

92. The signs of severe dehydration among infant are:


a. Drowsiness, depressed fontanels and decreased skin turgor
b. Anxiety and increased skin turgor
c. Drowsiness, bulging fontanels and decreased skin turgor
d. Excessive crying and excessive thirst
Ans : a

93. Skin is a part of which system in the body?


a. Endocrine b. Integumentary
c.Respiratory d. Circulatory
Ans : b

94. The strategy adopted to ensure quality nursing care by statutory body is :
a. Accreditation b. Re- registration
c.Licensure & registration d. Licensure
Ans : c

95. Most common strain of E. coli giving rise to traveler diarrhea is :


a. Entero-pathogenic b. Entero –toxigenic
c.Entero –aggregative d. Entero – invasive
Ans : b

96. Down syndrome among newborn is due to:


a. Chromosomal error b. Nutritional deficiency
c.ABO incompatibility d. Inborn error of metabolism
Ans : a

97. The difference between oral and axillary temperature is:


a. 1 degree Celsius b. 1.6 degree Celsius
c.0.6 degree Celsius d. 3 degree Celsius
Ans : c

98. While documenting the high pulse rate of a patient, nurse must document it as?
a. Tachycardia b. Hyperpyrexia
c.Arrhyhmia d. Tachypnea
Ans : a

99. The number of pulse beats per minute is known as:


a. Tension b. Rhythm
c.Rate d. Volume
Ans : c

100. Which of the following is the best answer to practice leadership?


a. Laissez faire b. Transactional leadership
c.Democratic leadership d. Autocratic leadership
Ans : b

101. Which of the following factors leads to growth of cities?


a. Industrialization b. Housing facility
c.Fertility of land d. Natural calamities
Ans : a

102. Which safety measure will you take for a drowsy patient?
a. Keep the side rails up
b. Keep unnecessary furniture away
c. Keep the lights on
d. Keep extra linen at bedside
Ans : a

103. The nurse act as an advocate when she does one of the following
a. She postpones the physical assessment until the client is calm
b. She encourages the client to express feelings
c. She assesses the client for injury
d. Ask the relatives to give their consent instead of patient
Ans : d

104. The thermoregulation center is located at:


a. Hypothalamus b. Cerebrum
c.Pituitary d. Medulla
Ans : a

105. While performing oral hygiene to the unconscious, which additional article should
be kept in the mouth care try?
a. Disposable spatula b. Laryngoscope
c.Mouth gag d. Artery forceps
Ans: c

106. Which of the following is not a type of communication?


a. Non – verbal & verbal b. Message
c.Surcastic d. Telecommunication & internet
Ans : b

107. When you come across a person lying unresponsive on the road, your first action
should be to?
a. Make the person to lie on one side
b. Call an ambulance and start CPR
c. Make the person to sit
d. Leave the person wake up himself
Ans : b
108. The best method for assessing respiration in a non-responsive person is:
a. Put hand front of nose b. Look listen and feel
c.Put the hand on the chest d. Hear heartbeat
Ans : b

109. The highest level of need in Maslow’s hierarchy is:


a. Self – esteem b. Physiological
c.Self – actualization d. Safety
Ans : c

110. What is true regarding H. pylori infection?


a. Bacteria b. Toxin producing microbe
c.Rota virus d. Immunoglobulin
Ans : a

111. Which of the following food stuff is a good source of protein ?


a. Milk and milk products b. Green leafy vegetables
c.Fruits d. Cereals
Ans : a

112. Which of the following techniques is used in psychotherapy?


a. Narcotic need b. Projective
c.Electroconvulsive therapy d. Dream analysis
Ans : d

113. Exchange of gases takes place in which part of the body?


a. Liver b. Kidney
c.Lung d. Heart
Ans : c

114. Duration of 3rd stage of labor is:


a. 30-45 minutes b. 1 hour
c.12 hours d. 1.5 hours
Ans : a

115. Which of the following are not approaches of health education?


a. Incentive approach b. Service approach
c.Education approach d. Regulatory approach
Ans : a
116. The significant sign of pyloric stenosis is:
a. Frequent vomiting b. Projectile vomiting
c.Blood in vomit d. Bile in the vomit
Ans : b

117. Which statutory body is responsible for ensuring the standards of nursing education
in India?
a. India Nursing Council b. NAAC
C.State Nursing Council d. University Grants Commission
Ans : a

118. Who is revered as the founder of modern nursing?


a. Betty Newman b. Jean Watson
c.Florence Nightingale d. Virginia Henderson
Ans : c

119. Which one of the following is not a feature of health education?


a. Knowledge and skill acquired actively
b. Aims at impulsive actions
c. Disciplines primitive desire
d. Develop self- expression
Ans : b

120. Helper cells belongs to


a. Macrocytic b. Macrophages
c.T cells d. B cells
Ans : c

GMCH Chandigarh 2016


1. The process of precisely destroying cancer tissue in the brain by using:
a. Photodynamic therapy b. Cryosurgery
c.Gamma Knife surgery d. Robotic surgery
Ans : c

2. The Lung Volume/Capacity that does not change with pulmonary disease is:
a. Vital capacity b. Total lung capacity
c.Tidal volume d. Inspiratory capacity
Ans : b

3. As a precautionary measure, you will perform the following test before taking a
sample for arterial blood gas analysis:
a. Homan’s test b. Allen’s test
c.Adson’s test d. Trendelenberg’s test
Ans : b

4. The device that can deliver oxygen at concentration of 90-100% is the:


a. Venture mask b. Partial rebreathing mask
c.Non rebreathing mask d. Nasal cannula
Ans : a

5. The goal of pursed lip breathing exercise is to :


a. Prevent aspiration pneumonia
b. Strengthen the diaphragm
c. Loosen the bronchial secretions
d. Reduce the amount of trapped air
Ans : d

6. While performing endotracheal suction in an adult, you will ensure that you:
a. Apply suction for at least 20-30 seconds at a time
b. Apply suction for only 10-15 seconds at a time
c. Set the wall – unit suction pressure at 40-60 mm Hg for adequate suctioning
d. Set the wall- unit suction pressure at 180-200 mm Hg for adequate suctioning
Ans : b

7. A patient admitted in the emergency is having tachypnea with respiratory rate of 28


breaths per minute. The likely diagnosis is:
a. Narcotic overdose b. Raised intracranial pressure
c.Pneumonia d. Brain injury
Ans : c

8. On auscultation of a breathless patient you hear crackling sounds at the lung bases.
The most likely diagnosis is :
a. Pneumothorax b. Pulmonary edema
c.Bronchial asthma d. Emphysema
Ans : b

9. A patient having a pneumothorax is being treated with a chest to and underwater


seal drain. The water level in the drain is oscillating rhythmically with respiration
and there is intermittent bubbling. This means that:
a. The tube is patient and the lung has fully expanded
b. The tube is patient but the lung has not fully expanded
c. The tube blocked and the lung has expanded
d. The tube is displaced and there is surgical emphysema
Ans : b

10. The Arterial Blood Gas that is most suggestive of respiratory acidosis is:
a. Ph7.3,CO2 50mm Hg b. Ph7.5,CO2mmHg
c.Ph7.3,HCO3 12mmol/L D. Ph7.5,HCO3 30mmol/L
Ans : a

11. A coronary Artery Disease patient in the Cardiac Care Unit complain of sudden onset
of breathlessness. Blood pressure is 160 mmHg. The first nursing action you will
perform is:
a. Administer intravenous morphine to reduce anxiety
b. Prop up the patient in high –Fowler position
c. Send an arterial blood gas sample
d. Evacuate the patients from the ward
Ans : b

12. A fire breaks out in your ward. The first action you take is :
a. Call 101 Fire Brigade
b. Shut off the oxygen supply
c. Do use the fire with the fire extinguisher
d. Evacuate the patients from the ward
Ans : a
13. While checking the blood pressure in the upper limb of a patient with a
sphygmomanometer, you observe that she has developed carpalspasm. The most
likely cause is:
a. Hypokalemia b. Hyponatremia
c.Hypocalcemia d. Hyperphosphatemia
Ans : c

14. The following statement is true regarding transfusion of blood to a


hemodynamically stable adult patient:
a. 150 mL should be transfused in the first 15 minutes
b. Blood should be administered slowly and completed not before 6 hours
c. The transfusion should be completed within 4 hours
d. The unit of blood should be warmed at the bedside for one hour before starting
transfusion
Ans : c

15. The initial energy selection for defibrillation in pulseless ventricular tachycardia fir
a:
a. Monophasic defibrillator is 32 J
b. Biphasic defibrillator is 16 J
c. Monophasic defibrillator is 100 J
d. Biphasic defibrillator is 150 J
Ans : d

16. In a 78- year –old smoker with streptococcal pneumonia and difficulty ‘ in coughing
out secretions, the nursing diagnosis is:
a. Community Acquired Pneumonia with cough
b. Pneumonia in an elderly smoker who needs chest physiotherapy
c. Acute Chest Infection with weakness of chest muscles
d. Ineffective airway clearance related to effects of pneumonia
Ans : d

17. A patient passes urine as given in the sequence: Tuesday 8 AM 200 ml 12 noon 250
ml,3 PM 300ml,6 PM 300ml,9PM 250 nil, Wednesday 3 AM 200ml,8 AM 300 ml. The
24 hour 8 AM collection of urine for laboratory evaluation is quantified to be:
a. 1350 ml b. 1500ml
c.1600 ml d. 1800 ml
Ans : c
18. The badge that is applied over an amputated knee stump while preparation for a
prosthesis usually a:
a. Spiral bandage b. Figure of eight bandage
c.Recurrent bandage d. Circular bandage
Ans : b

19. A patients to the emergency with a history of multiple bee sings and a blood
pressure of 80/50 mm Hg. He is having:
a. Hypovolemic shock` b. Spinal Shock
c.Hemorrhagic shock d. Anaphylactic shock
Ans : d

20. The patient in the gastroenterology ward is on nasogastric tube with continuous
low-pressure suction, the acid-base disorder you anticipate this sitting is:
a. Metabolic acidosis b. Metabolic alkalosis
c.Respiratory acidosis d. Respiratory alkalosis
Ans : b

21. While caring for a patient of Bronchial Asthma, you identify that the patients
respiratory condition has become critically worse when:
a. There is an increase in the intensity of wheezing
b. There is a decrease in breath sounds all over the chest
c. Crepitations and wheezing are both present
d. The patient says, “I am feeling more breathless than when I come to hospital”
Ans : b

22. You are teaching Kegel exercises to a female patient. Her understanding of exercises
is correct when she sates that:
a. These exercises will reduce dribbling of my urine
b. These exercises will reduce the recurrent urinary tract infections I get
c. These exercises are for treating my hemorrhoids
d. These exercises will reduce fecal incontinence
Ans : a

23. The MAST garment is used for treatment of :


a. Acute respiratory distress(B)
b. Hemorrhagic Shock
c. Incontinence of urine
d. Paraplegia
Ans : b
24. The common abdominal incision for gallbladder surgery is:
a. McBurney’s Gridiron b. Kocher’s incision
c.Pfannenstcil incision d. Left paramedian incision
Ans : b

25. You are assessing a patient of acute pancreatitis for the complication of paralytic.
The key symptom indicating this complication is:
a. Recurrent vomiting b. Abdominal pain
c.Inability to pass flatus d. Watery Diarrhoea
Ans : c

26. In General anesthesia, the stage of excitement and dreams is :


a. Stage I b. Stage II
C.Stage III d. Stage IV
Ans : b

27. The post-operative exercise of “huffing” is used primarily to help the patient:
a. Make oxygen reach the alveoli
b. Strengthen the chest muscles
c. Reduce lung collapse
d. Loosen the chest secretions
Ans : d

28. While dressing the abdominal wound on the 51th post-operative day, you observe
that some intestinal loops are protruding outside. This is called:
a. Perforation b. Wound dehiscence
c.Evisceration d. Intestinal hernia
Ans : c

29. A high frequency chest wall oscillation (HFCWO) vest is used for:
a. Providing high frequency ventilation
b. Loosening thick chest secretions
c. Preventing pulmonary embolism
d. Providing Cardio-pulmonary resuscitation
Ans : b

30. A patient has a pressure ulcer in which subcutaneous fat is visible BUT bone, tendon
or muscle is not exposed or visible. This is a :
a. Stage II ulcer b. Stage III ulcer
c.Stage IV ulcer d. Unblanchable ulcer
Ans : b

31. An example of an Open Drainage system used in surgical wounds is:


a. Jackson-Pratt drain b. Hemovac
c.T-Tube biliary drain d. Penrose Drain
Ans : d

32. Paradoxical respiration is most commonly seen in:


a. Congestive heart failure b. Obstructive airway disease
c.Cardiac tamponade d. Flail chest
Ans : d

33. A patient with classical heat stroke will have:


a. Dry Skin b. Profuse sweating
c.Cool Skin d. Normal body temperature
Ans : a

34. The logroll turning method is used in:


a. Lung abscess b. Cerebrovascular accident
c.Spinal injury d. Epilepsy
Ans : c

35. A body temperature of 390 centigrade is equivalent to a Fahrenheit (in nearest


degrees):
a. 99 b. 100
c.101 d. 102
Ans : d

36. An example of a cervical skeletal traction device is:


a. Halo device b. Buck’s traction
c.Thomas splint d. Cramer wire splint
Ans : a

37. You tell a 65 years old patient ,”Listen carefully and repeat after me: Apple, Pen,
Table.” If he repeats these three words immediately after you in the correct
sequence, he has normal:
a. Judgement b. Orientation
c.Recall d. Registration
Ans : d
38. You observe that a patient of head injury opens his eyes on painful stimuli but not
on command; he localizes the site of the painful stimulus; and mutters
incomprehensible sounds. His Glasgow Coma Scale Score is in the range of :
a. 5-6 b. 9-10
c.12-13 d. 14-15
Ans : b

39. A DEXA Scan is used in the diagnosis of:


a. Breast cancer b. Osteoporosis
c.Coronary artery disease b. Pulmonary embolism
Ans : b

40. A patient has burns over the front of the chest and abdomen. The percentage of
burns is calculated to be:
a. 9% b. 18%
c.24% d. 36%
Ans : b

41. The Mallampati Classification deals with:


a. Pattern of sleep disturbance in insomnia
b. Wound assessment in unstageable ulcer
c. Airway assessment in anesthesiology
d. Risk of Health-care associated infections in intensive care settings
Ans : c

42. The ideal angle of insertion of a needle for administering an intra-dermal injection is
:
a. 10 degrees b. 30 degrees
c.45 degrees d. 60 degree
Ans : a

43. The Z- track technique of intramuscular injection is used to:


a. Avoid nerves and veins in the path
b. Reduce backward leakage of drug
c. Reduce the pain of injection
d. Reduce bleeding and hematoma
Ans : b
44. During an Intravenous (IV) infusion of saline, you observe that the infusion has
stopped and there is mild swelling around the IV site. The skin is cool and puffy. This
suggests:
a. Infiltration b. Phlebitis
c.Air Embolism d. Atherosclerosis
Ans : a

45. The last menstrual period of a pregnant lady started on 20th February 2016. Her
expected date of delivery is:
a. 07th November 2016 b. 14th November 2016
c.20th November 2016 d. 27th November 2016
Ans : d

46. You are required to give 1000 ml of Normal Saline over 10 hours using a micro-drip
infusion set. You will adjust the drop rate (in micro-drips per minute calculated to
the nearest 10 micro-drops)at:
a. 200 b. 150
c.100 d. 50
Ans : c

47. An infusion bag of dobutamine contains 250 milligram/205milliliter. It is to be


infused at the rate of 3 microgram /kg/minute to a 70 kg man. The rate of infusion
in milliliter/hour(calculated to the nearest milliliter ) will be:
a. 32 b. 26
c.21 d. 13
Ans : d

48. A patient with the following infection should be assigned to a Negative –pressure
isolation room:
a. Laryngeal tuberculosis b. Herpes simplex
c.Mycoplasma pneumonia d. Impetigo
Ans : a

49. A patient is on Oral anticoagulant (Warfarin) for treatment of pulmonary embolism.


You will monitor the adequacy of anticoagulation by:
a. Modified Ivy Test
b. Activated partial Thromboplastin Time (APIT)
c. Indirect Coomb’s Test (ICT)
d. International Normalized Ratio (INR)
Ans : d
50. A patient in your ward develops a generalized tonic – clonic seizure. The
appropriate immediate action you take is:
a. Restrain the patients arms and legs to prevent injury
b. Insert a spoon in his mouth to prevent tongue bite
c. Turn the patient on the side
d. Pinch the patient’s nose to stop the seizure
Ans : c

51. One of the following Singular: Plural pair correct:


a. Man-of-War : Mans-of-wars
b. Commander –in chief : Commander-in -chiefs
c. Son –in-Law: Sons-in-law
d. Passer-by: Passer-byes
Ans : c

52. The synonym of the word EXILE is:


a. Exonerate b. Escalate
c.Abscond d. Deport
ans : d

53. In usage of adjectives, the correct formation of the sentence is:


a. Ram is old than Shyam b. Ram is older than Shyam
c. Ram is oldest than Shyam d. Older Ram is than is Shyam
Ans : b

54. In Grammar, the sentence “The Sun rises in the east” is:
a. Present continuous tense b. Simple present tense
c.Past continuous tense d. Simple past tense
Ans : b

55. The following is an example of Direct Speech:


a. Ram said he need a clean shirt.
b. Did Ram need a clean shirt?
c. A clean shirt was needed by Ram.
d. Ram said. “I need a clean shirt”
Ans : d

56. Identify the sentence which has correct punctuation:


a. What is your name b. The scenery is beautiful?
c.Help ! Ihave been robbed. d. Do you belong to this college.
Ans : c

57. The synonym of “ conceited” is:


a. Humble b. Timid
c.Rich d. Proud
Ans : d

58. To have” an albatross around your neck” means:


a. To be very rich by birth b. To be sentenced to death
c.To carry a difficult burden d. To have enemies around you
Ans : c

59. Artificial methods of birth control are not acceptable in:


a. Buddhism b. Roman Catholics
c.Protestants d. Hinduism
Ans : b

60. A nurse who conducts an illegal abortion has-committed:


a. An act of omission b. An act of incompetence
c.Unintentional tort d. An act of commission
Ans : d

61. In the emergency you have 4 patients. By prioritizing your management plan, which
patient’s problem will you address first?:
a. Forearm in a plaster cast for 24 hours with numbness in fingers of the same
hand
b. Chronic Asthmatic: Mild Cough, Respiratory rate 22/minute, Moderate wheezing
c. Dengue patient with temperature 1010F, BP 100/60 mmHg, platelet count
40,000/mm3
d. Cardiac patient on Aspirin with mild bleeding from gums
Ans : a

62. In a disaster, you will prioritize your management and attend first to the patient
who is:
a. In severe pain due to fracture of the tibia
b. In deep coma with massive head injury
c. Having breathlessness with flail chest
d. Conscious patient with a hematoma on the scalp
Ans : c
63. One of the primary function of a Circulation Nurse in the Operation Theatre is to:
a. Verify that the informed consent has been taken
b. Prepare the surgical sterile table and instruments
c. Assist the surgeon during the surgery
d. Set up the laparoscope/other instruments for the surgeon
Ans : a

64. In communicating with patients, the word ”rapport” means:


a. To discuss problems and treatment with patient
b. To report the patients feelings to a doctor
c. To understand the patients language
d. To show genuine feeling of goodwill
Ans : d

65. The use of “touch” to convey your feeling to a patient is called:


a. Extrasensory perception b. Haptic communication
c.Gestalt therapy d. Neural networking
Ans : b

66. The word URL in computer language means:


a. Universal Recognizable Language
b. Universal Resource Locator
c. Unlimited Resource Locator
d. Uniform Recognized Language
Ans : a

67. In a Microsoft Word Document, you can change from “Times New Roman” to ”Arial”
by going to the:
a. Clipboard section b. Paragraph section
c.Editing section d. Font section
Ans : d

68. In the web address, which of the following indicate that a website is secure:
a. http:// b. https://
c.httpsw:// c. httpsec://
Ans : b

69. In conducting a search on the internet, the use of the words AND,OR,NOT indicate:
a. Hypertext Machine Language b. Truncation Symbols
c.Wildcard Symbols d. Boolean Operators
Ans : d

70. You send an e-mail as follows: To: Tom, Cc: Dick, Harry, Bcc: jack, Jill. The following
statement is true:
a. Tom knows that a copy has been sent to Disk and Harry
b. Tom knows that a copy has been sent to Jack, Jill, Disk and Harry
c. Jack knows that a copy has been sent to Jill
d. Disk and Harry know that a copy has been sent to Jack and Jill
Ans : a

71. The nursing theory framework model of Florence Nightingale was:


a. Independent-functioning b. Self-care
c.Adaptation d. Natural –healing
Ans : d

72. The best example of “subjective data” from the options given below is:
a. The patient is having chest discomfort for 2 hours
b. The patient’s temperature was 990 F at 5.00PM
c. The patient has 5cm Stage 2 pressure sore
d. The patient has a blood pressure of 120/80mm Hg in supine position
Ans : a

73. The “independent action” performed by a nurse that is administratively acceptable


is when she:
a. Administers injection Morphine to relieve patient’s acute abdominal pain
b. Discharge a patient on his request
c. Gives cold sponging to a febrile patient
d. Orders an ultrasound for a patient with abdominal pain
Ans : c

74. A patient diagnosed with a terminal illness tells the doctor, Life is not fair. This
cannot happen to me”. This is an expression of:
a. Denial b. Bargaining
c.Introspection d. Acceptance
Ans : a

75. A hospice facility has the following principles:


a. Death is not acceptable and the process of dying must be prolonged
b. Pain relief is a priority in terminally ill patients
c. Bereavement care for family is not a part of this form of care
d. The focus is on comfort of the dying patient and the family
Ans : d

76. In a healthy 70 kg adult male, the intracellular fluid volume


a. 42 Liters b. 28 Liters
c.14 Liters d. 3.5 Liters
Ans : b

77. The daily insensible water loss from the body normally is (in milliliter, Ml)
a. 100-200ml b. 400-800ml
c.1200-1400ml d. 1600-1800 ml
Ans : b

78. On the basis of Body Mass Index (BMI), a person with a height of 150 cm and weight
of 70 kg is defined to be:
a. Obese b. Overweight
c.Normal Weight d. Underweight
Ans : b

79. The transfer of antibodies from mother’s milk to the baby is called:
a. Artificially acquired passive immunity
b. Artificially acquired active immunity
c. Naturally acquired passive immunity
d. Naturally acquired active immunity
Ans : c

80. In preparation for abdominal surgery, a patient has to be administered a pre-


operative enema. The position of the patient should be:
a. Lithotomy position b. Left lateral Sim’s position
c.Trendelenberg position d. Semi-Fowler position
Ans : b

81. A patient with fracture of left tibia is using crutches for walking . A complication as a
result of improper use of crutches for which regular assessment should be done is:
a. Venous thrombosis b. Foot drop
c.Wrist drop d. Cervical spondylosis
Ans : c
82. While monitoring a patient of Myocardial Infarction in your Coronary Care Unit you
observe an isolated ventricular ectopic on the monitor. You will inform the
physician and:
a. Continue to monitor the patient
b. Prepare to defibrillate the patient
c. Prepare to administer intravenous Lignocaine
d. Prepare to administer intravenous Nitroglycerin
Ans: a

83. A patient who is prescribed oral cyclosporine should be advised to avoid:


a. Meat b. Eggs
c.Grapefruit Juice d. Green leafy leafy vegetables
Ans : c

84. A 40 year old patient is admitted with crushing chest pain and an Electrocardiogram
(ECG) showing Acute Myocardial Infarction. He is pale, has Blood pressure of
90/60mm Hg, irregular pulse of 120 beats per minute, respiratory rate 28/minute.
The correct statement regarding nursing care plans is:
a. Nursing diagnosis :Acute Myocardial Infarction with Shock
b. Nursing diagnosis: chest pain with complications, under evaluation
c. Long-term Goal: ECG will be normal at the time of discharge
d. Short –term Goal: within 24 hours patient will state that pain has decreased
Ans : d

85. An example of an aerobic exercise is:


a. Running 100 meters at a fast pace(B)
b. Jogging slowly for 2 kilometers
c. Lifting 50 kg weights 10 times
d. Doing yoga by taking deep and slow breaths, from alters nostrils
Ans : b

86. The osmolality of 0.45% Sodium Chloride (NaCl) is:


a. 154mOsm/L b. 278mOsm/L
c.308mOsm/L d. 512mOsm/L
Ans : a

87. The suture material that is both absorbable and synthetic is:
a. Polyamide b. Polypropylene
c.Polyglycolic acid d. Chromic catgut
Ans : c
88. You identify that a poly-trauma patient has gone into Acute Respiratory Distress
Syndrome when:
a. He develops pallor and cyanosis
b. You hear crackling sounds over the chest on auscultation
c. His respiratory rate goes up from 18/minute to 30/minute
d. His blood pressure falls from 140/80 mm Hg to 90/60 mmHg
Ans : c

89. The volume of air normally inhaled and exhaled with each regular breath is called:
a. Vital capacity b. Tidal volume
c.Timed vital capacity d. Anatomical dead space
Ans : b

90. A Guedel airway is a :


a. Nasopharyngeal airway b. Tracheostomy tube
c.Oropharyngeal airway d. Laryngeal mask airway
Ans : c

Tamil Nadu PSC for Maternal and


Child Health Officer Nursing 2015
1. The function of B lymphocyte is
a. provide cell mediated immunity
b. Humoral immunity
c. Rejection of transplanted organs
d. Removal of antigens in a inflammation
Ans : b

2. Which molecule cannot cross the blood-brain barrier?


a. Glucose b. Protein
c.Potassium d. Urea
Ans : d

3. The glomerular Filtration Rate ( GFR) of normal healthy adult is


a. 50ml/min b. 100ml/min
c.120ml/min d. 180ml/min
Ans : c

4. Which of the following is unmatched according to their sensory functions?


a. Tongue and nose –Chemosensory
b. Ears –Organ of corti
c. Nose –Olfactory nerve
d. Eyes –Maxillary nerve
Ans : d

5. Which of the following is the precursor of Vitamin D?


(a) Glycolipid (b) Cholesterol
( c ) Ergosterol (d) Cephalins
a. A. is correct but not (b), ( c ) and (d)
b. B. is correct but not (a),( c ) and (d)
c. C. and B are incorrect
d. D. is correct but not (a) ,(B ) and ( c )
Ans : b

6. Find the odd one out in case of co-factors and metabolic factors of enzymes?
a. Zn 2+ b. Mg2+
b. CoA D. Mo
Ans : c

7. Which of the following hormones promotes the glycogen metabolism?


a. Epinephrine b. Non-epinephrine
c.Glucocorticoids d. Mineralocorticoids
Ans : a

8. Match the list of symptoms with the list of particular electrolyte imbalance:
List I List II
(a) Hypocalcemia 1. Paralysis
(b) Hypomagnesemia 2. Diarrhea
(c) Hypokalemia 3. Leg and foot cramp
(d) Hyperkalemia 4. Tetany
(a) (b) ( c ) (d)
a. 2 3 1 4
b. 3 1 4 2
c. 3 1 4 2
d. 4 3 1 2
Ans : d

9. Match it.
Wound drainage Color and Consistency
(a) Serous 1. Red with blood, thin
( b) Sanguineous 2. Clear ,light yellow ,thin
( c )Sero Sanguineous 3. Creamy, yellow, green, white
( d) Purulent 4. Pink to light red
( a) (b) (c ) (d)
a. 2 1 4 3
b. 1 2 3 4
c. 4 3 2 1
d. 1 3 2 4
Ans : a

10. What is level of raise in the body temperature during ovulation?


a. 10F b. 50C
c.30F d. 20C
Ans : a
11. The reasons for using the parenteral route to administering medication include
a. Parenteral Medication lasts longer than an oral medication
b. It is the least expensive method
c. It is easier to measure an curate dose
d. The parenteral route allows more rapid, absorption than the oral route
Ans : d

12. The left ureter is slightly higher than the right ureter because the
a. Left kidney is higher than right
b. Right kidney is higher than right
c. Left kidney performs more functions
d. Left ureter has a three layered wall
Ans : a

13. Match the following :


I II
(a) Fecal Incontinence 1. Cramping and Hyperactive bowel sounds
(b) Fecal Impaction 2. Involuntary elimination of feces
( c) Flatulence 3. Large, hard, stool in folds of rectum
(d)Diarrhea 4. Accumulation of gas in GI tract
(a) (b) (c) (d)
a. 4 3 2 1
b. 3 2 4 1
c. 1 2 3 4
d. 2 3 4 1
Ans : d

14. Which part of the brain regulates NREM sleep?


a. Pons b. Hypothalamus
c. Basal forebrain d. Amygdale
Ans : c

15. While instilling eye drops, instruct your patient to


a. Look down and away
b. Look up and away
c. Look straight a head
d. Look up and directly at the dropper
Ans : b

16. Which gait should you teach a patient who can bear weight on both legs?
a. Three point b. Four point
c.Two point d. Five point
Ans : b

17. The term ‘Nurture’ refers to the effect of one of the following on human
development
a. Heredity b. Environment
c.Food and Safety d. Rewards
Ans : b

18. The person who harbors the pathogen without suffering from any ill effect of that is
called
a. Source b. Latent infection
c.Host d. Healthy carrier
Ans : d

19. The composition of Pentavalent vaccine is


a. Diphtheria , pertussis, Tetanus, Cholera and OPV Measles, Mumps, Rubella,
Typhoid and Cholera
b. Diphtheria, Pertussis, Tetanus,Hemophilus influenza and Hepatitis B
c. Typhoid ,Chickenpox Measles Rubella and Hepatitis B
Ans : c

20. The condition where bacteria circulate and multiply in the blood is called
a. Bacteremia b. Pyremia
c.Septicemia d. Fibrinolysis
Ans : c

21. Which one is not a main system files of MS DOS?


a. IO sys b. COMMAND. Sys
c.MSDOS. sys d. COMMAND. COM
Ans : b

22. Which of the following is related to Fifth Generation?


(a) Company device based on artificial intelligence
(b) Very large scale integrated
a. Only A is the correct answer
b. Only B is the correct answer
c. Both A and B are correct
d. Both A and B are wrong answers
Ans : a

23. Which one of the following is not a statistical package


a. EPI-INFO b. MICROSIRIS
c.LISP-STAT d. SSI
Ans : d

24. The shortcut key used to insert a new slide in MS Power point is
a. Ctrl+M b. Ctrl+N
c.Ctrl+Q d. Ctrl+S
Ans : a

25. What are the responsibilities of a circulatory nurse?


I. Verify consent
II. Ensure proper temperative, humidity, lighting and safe functioning of
equipment
III. Monitory aseptic practices
IV. Responsible for ensuring the second verification of surgical procedure and
site
a. 1,2 and 3 b. 2,3 and 4
c.1,3 and 4 d. 1,2,3 and 4
Ans : d

26. Which of the ones wrongly matched:


I. Hiatal hernia - Herniation of the portion of the stomach area
oesophagus
II. Achalasia - Foul smelling breath
III. Zeuker’s diverticula - Pharyngeal pouches
IV. Haliton’s - Absent peristalsis of distal esophagus
a. I and II b. II and III
c.II and IV d. III and IV
Ans : c

27. The following are the complications of oxygen administration


I. It supports combustion II. Causes CO2 Narcosis
III.Oxygen toxicity IV. Absorption atelectasis
a. I,II and III b. II,III and IV
c.I, II and IV d. I,II,III and IV
Ans : d

28. A patient with hepatic encephalopathy is prescribed lactulose. The nurse knows that
purpose of this medication is to
a. Decrease the gastric acidity
b. Prevent reabsorption of Na+ and water
c. Acidification of feces and trapping of ammonia causing its elimination in feces
d. Decrease the bacterial flora, decreasing formation of ammonia
Ans : c
29. Reed Sternberg cell is the Hallmark in which disease
a. Hodgkin’s disease b. Non-Hodgkin’s Lymphoma
c.Multiple Myeloma d. Burkett’s Lymphoma
Ans : a

30. The clinical features of Portal Hypertension are the following


1. Splenomegaly 2. Caput medusa
3.Fruit breath odour 4. Thrombocytopenia
a. 1,2 and 3 b. a, b, c and d
Ans : c

31. Urge incontinence, incomplete bladder emptying. Bladder Sphincter incoordination


are the features of which days functions
a. Atonic Bladder b. . Hypertonic Bladder
c. Flaccid Bladder d. Cortical Lesion
Ans : b

32. Match List with List II and select the correct answer using the codes given below the
lists:
List I List II
(a). Rhonchi 1. High pitched squeaking or musical sound
(b). Wheezes 2. Discontinuous, high –pitched sounds
(c). Stridor 3. Snoring or rattling sounds from
obstruction of Large airways
(d). Crackles 4. Musical or crowing sound from partial
obstruction of Larynx &Trachea
(a) (b) (c) (d)
a. 3 1 4 2
b. 1 2 4 3
c. 2 4 3 1
d. 4 2 3 1
Ans : a

33. Which of the statements given below is/are true about Antigen – specific
immunoglobulin E (IgE) in allergic Rhinitis?
a. Releases histamine and prostaglandins
b. Maintains the inflammatory response
c. Produced as a result of initial exposure to an allergen
d. Causes infiltration of inflammatory cells into the nasal tissue
Ans : c
34. An immediate interventions during an injury to muscle or joint include the following
I. Rest to affected part
II. Applying ice compress to the injured area
III. Compressing the involved extremity
IV. Elevating the extremity
a. I,II and III b. II,III and IV
c. I,III and IV d. I,II,III and IV
Ans : d

35. Hemophilia is a ……………………. disorder.


a. X linked dominant b. X linked recessive
c.Autosomal dominant d. Autosomal recessive
Ans : b

36. In which condition the number of eosinophils is increased ?


a. Allergic conditions
b. Increase in the level of calcium
c. Decrease in the level of sodium’
d. Increase in the level of potassium]
Ans : a

37. Aspirin prolongs bleeding time by inhibiting the synthesis of


a. Clotting factors in liver
b. Prostacyclin in vascular endothelium
c. Cyclic AMP in platelets
d. Thromboxane A2 in platelets
Ans : d

38. Match the following :


Part I Part II
(a) Live Vaccine 1. Typhoid
(b) Killed Vaccine 2. Rubella
(c) Toxoids 3. Rabies
(d) Antisera 4. Diphtheria
(a) (b) (c) (d)
a. 2 1 4 3
b. 4 2 3 1
c. 3 4 2 1
d. 2 4 1 3
Ans : a

39. Vande Mataram Scheme is an integral part of


a. 20 point programme
b. Minimum needs programme
c. RCH programme
d. Nutritional programme
Ans : c

40. NRHM was launched in the year of


a. 2004 b. 2005
c.2006 d. 2007
Ans : b

41. The occupational disease caused by inhalation of sugar – cane dust is


a. Silicosis b. Anthracosis
c.Byssinosis d. Bagassosis
Ans : d

42. Primordial prevention means prevention of


a. Infections b. Complications
c.Diseases d. Risk factors
Ans : d

43. The following are the components of MCH EXCEPT


a. Occupational health
b. Care of mentally retarded children
c. School health
d. Care of children in special settings
Ans : b

44. Cornea and conjunctiva will become dry in


a. Keratomalacia b. Bitot’s Spot
c.Xerophthalmia d. Night Blindness
Ans : c

45. Arrange the demographic cycle


I. High stationary stage II. Early expanding stage
III.Late expanding stage IV. Low stationary stage
V. Decline stage
a. I, II, III, IV, V b.I, III, II, IV ,V
c.I, V, IV, III, II d. IV, III, II, I, V
Ans : a

46. Malaria is a
a. Zoonotic Disease b. Viral Disease
c.Bacterial Disease d. Protozoal Disease
Ans : d

47. Point out the wrong one in the following measurements in epidemiology
a. Measurement of mortality
b. Measurement of disability
c. Measurement of nurse –patient ratio
d. Measurement of natality
Ans : c

48. Find the odd one in the following response. Following are the examples of live
vaccines
a. BCG b. Measles
c.Oral Polio d. Tetanus
Ans : d

49. An effective anti TB drug against intracellular as well as extracellular bacilli is,
a. INH b. Streptomycin
c.Rifampicin d. Pyrazinamide
Ans : c

50. The teaching directed towards creating/promoting an interest in learning is known


as
a. Principle of creativity b. principle of motivation
c.Principle of training d. Principle of reinforcement
Ans : b

51. Major approaches in problem solving are


a. Inductive, Deductive, Active Analytic
b. Inductive, Deductive, Active Synthetic
c. Inductive, Deductive, Initiative, Synthetic
d. Inductive, Deductive, Purposive,, Synthetic
Ans : b

52. Arrange the stages of team growth


a. Performing, Norming, Storming and Forming
b. Forming ,Storming, Norming and performing
c. Storing, Norming, Forming and performing
d. Norming, Forming, Performing and Storming
Ans : b

53. Normal coping strategies are activated in the following phase of crisis
a. Phase I-Establishing relationship
b. Phase II-Assessment
c. Phase IV-Intervention
d. Phase V-Termination
Ans : a

54. OSCE means


a. Objective Structured Clinical Examination
b. Observation Structured Clinical Examination
c. Objective Subjective Clinical Examination
d. Observation Subject Course Examination
Ans : a

55. A student used the formula to calculate the daily protein requirements of a patient.
It describes the following category of cognitive domain.
a. Comprehension b. Application
c.Analysis d. Synthesis
Ans : b

56. What is the standard method of removal of impacted cerumen from the external
canal?
a. Irrigation b. Suction
c.Instrumentation d. Instillation of oil drops
Ans : a

57. Which of the statements regarding epistaxis is not true?


a. Posterior source of epistaxis is Kisselbach’s plexes
b. Cauterization is by silver nitrate application
c. Nasal packing for 48 to 72 hrs
d. Maxillary or ethmoidal surgical artery ligation
Ans : a

58. The complications of pelvic inflammatory diseases are the following EXCEPT:
a. Fibroids b. Septic shock
c.Ectopic pregnancy d. Infertility
Ans : a

59. In the elderly, the respiratory system undergoes the following changes EXCEPT:
a. Decreased number of alveoli
b. Decreased chest wall size
c. Increased rigidity of trachea
d. Increasing cough
Ans : d

60. What is the diagnosis if the AB’s values are


Ph7.2 paCO2- HC3-24 pao2-65mm
55mm meq/L
a. Respiratory acidosis
b. Metabolic acidosis
c. Compensated resp. acidosis
d. Mixed acid-base
Ans : a

61. A man swallowed a safety pin which is stuck in the esophagus. Which one of the
following is the correct treatment measure?
a. Emergent endoscopy
b. Ask the man to drink clear until the repeat X-ray shows pin has moved down to
stomach
c. Discharge the patient with the instruction to check the stool for the elimination
of pin
d. Prepare and administer polyethylene glycol
Ans : a

62. Match list I with list II and select the correct answer using the codes given below the
lists.
List I List II
(a) Infant mortality rate 1. Deaths of children aged under 5years/1000
live births
(b) Perinatal Mortality rate 2. Deaths of children under 28 days
of birth/1000 live births
(c) Neonatal mortality rate 3. Late fetal and early neonatal
deaths/1000 live births
(d) Under five mortality rate 4. Deaths of children less than one year/
1000 live births
(a). (b) (c) (d)
a. 1 2 3 4
b. 2 3 4 1
c. 4 3 2 1
d. 3 2 4 1
Ans : c

63. Three remarkable components of Integrated Management and Childhood Illness


(IMNCI) are
1. Improvement of case management skills of health care providers
2. Provision of essential supplies
3. Integration of child health services
4. Optimization of family and community practices in relation to child health
a. 1,2,3 b. 2,3,4
c.1,3,4 d. 1,2,4
Ans : d

64. Spot the correct statement in terms of “Epispadias.” It is an abnormal urethral


opening on the
a. Ventral aspect of the penis
b. Urethral scrotal junction
c. Shaft of the penis
d. Dorsal aspect of the penis
Ans : d

65. The causative organism for Varicella is


a. Corynebacterium diphtherias
b. C.Tetani
c. Bordetella pertussis
d. Varicella zoster virus
Ans : d

66. Certain code drugs which can be administered through tube for children in an
emergency
a. Dopamine, Dobutamine, Adenosine, Atropine
b. Naloxone, Dopamine, Glucose, Adenosine
c. Glucose, Atropine, Ipecac, Charcoal
d. Lidocaine, Epinephrine, Atropine, Naloxone
Ans : d

67. A child with typical facial features such as low nasal bridge with short upturned
nose, flattened mid face, long philtrum with narrow upper lip is diagnosed to have
a. Fetal Alcohol syndrome b. Autism spectrum disorder
c.Down syndrome d. Tourette syndrome
Ans : a

68. “Universal children is day” November 14th was initiated by


a. WHO b. UNICEF
c.ICDS d. NGO
Ans : b

69. Identify the wrong answer behavioral problems of childhood includes


a. Breath –holding spell b. Temper tantrums
c.Thumb sucking d. Failure to thrive
Ans : d

70. Hook worms are otherwise termed as:


a. Oxyruriasis b. Ancylostomiasis
c.Ascariasis d. Teniasis
Ans : b

71. Hernia protrusion of the meninges through a midline defect in the posterior
vertebral arch is known as
a. Meningocele b. Spina bifida
c.Spina bifida occulta d. Menigomyelocele
Ans : a

72. The optimal timing for surgical correction of cleft-lip is


a. 2-3 months b. 6-7 months
c.11-12 months d. 14-15 months
Ans : a

73. Match the development stage with their psycho social development according to
Eric Ericksol
A B
(a) Infancy 1 1.Sense of industry
(b) School age 2. Sense of intimacy
(c) Early adolescence 3. Sense of trust
(d) Late adolescence 4. Sense of identify
(a) (b) (c) (d)
a. 3 2 1 4
b. 3 2 4 1
c. 3 1 4 2
d. 3 4 2 1
Ans : c

74. Which statement by a person with paranoid schizophrenia most clearly indicates
the antipsychotic medication is effective?
a. I used to hear scanty voices but now I don’t hear them anymore
b. My medicine is working fine. I’m not having any problems
c. Sometimes it’s hard for me to fall asleep, but I usually sleep all night
d. I think some of the staff members don’t like me. They’re mean to me
Ans : a

75. Following are the dysfunctional behaviors associated with substance abuse
1. Manipulation 2. Impulsiveness
3.Anger 4. Acceptance
a. 2,3,4 b. 1,2,3
c. 1,2,4 d. 4,3,1
Ans : b

76. The Misinterpretation of a real stimuli within the environment is called as


a. Hallucination b. Delusion
c.Illusion d. Obsession
Ans : c

77. A nurse suggests activities for a 7-yr old child with autistic disorder. Which activity
is most likely to engage this child?
a. Playing checkers with another child
b. Building with blocks alone
c. Playing kickball with a small group of children
d. Having a birthday party with 6 to 8 other children
Ans : b

78. In managing a client with pre-eclampsia rest is important for the following reason,
EXCEPT:
a. Increases renal blood flow
b. Increases uterine blood flow
c. Decreases blood pressure
d. Decreases placental blood flow
Ans : d

79. The process of alteration in the shape of the fore coming head while passing through
the resistant birth passage during labor is called as:
a. Caput succedaneum b. Cephal hematoma
c.Moulding d. Cranial injury
Ans : c

80. Which of the following instrument will be used in recording of uterine contractions
from various parts of the uterus?
a. Cardiotocography b. Tocodynamometer
c.Tocometry d. Topography
Ans : b

81. A newborn born for a HIV infected mother is treated with


a. Zidovudine syrup 2 mg/kg
b. Zidovudine syrup 1 mg/kg
c. Zidovudine syrup 3 mg/kg
d. Zidovudine syrup 4 mg/kg
Ans : a

82. The progressive measurement of the pregnancy and calculation of ggesational age
by height of the fundus is done by
a. Naegle’s rule b. Robert’s rule
c.Mc Donald’s rule d. Simson’s rule
Ans : c

83. Find out the correct pair regarding nerve injuries that occur during birth
a. Facial palsy - 7th cervical nerve root
b. Brachial palsy - 6th and 7th cervical nerve root
c. Erb’s palsy - 5th and 6th cervical nerve root
d. Klumpke’s palsy - 5th cervical nerve root
Ans : c

84. The most crucial when structural malformation can occur in the fetus is
a. Just before implantation
b. Between fertilization and implantation
c. Embryonic period
d. Fetal period
Ans : c

85. The instrument which is used to known the position and length of the uterine cavity
is called
a. Uterine curette b. Uterine sound
c.Uterine forceps d. Uterine retractor
Ans : b

86. Consider the statements:


Regarding the cardinal movements during normal labor and find out which is in
sequential order?
I. Engagement, descent, internal rotation, crowing, flexion extension
II. Engagement, descent, flexion, crowing, restitution, lateral flexion
III. Engagement, descent, internal rotation, crowing, extension
IV. Engagement, descent, flexion, internal rotation, crowing, extension
Restitution, external rotation, lateral flexion of the statement
a. I alone is correct b. I and II are correct
c.I,II and III are correct d.IV alone is correct
Ans : d
87. Match List I correctly with List II and select your answer using the codes given below.

List I List II

(a) Craniotomy 1. To enlarge the pelvic capacity


(b) Cleidotomy 2. Perforation on the fetal head
(c) Symphysiotomy 3. Reduction in the bulk of the shoulder
girdle
(a) (b) (c) (d)
a. 1 2 3 4
b. 3 4 2 1
c. 4 3 1 2
d. 2 4 1 3
Ans : d

88. women having their first pregnancy at or above the age of 30 years are called as
a. Grand b. Garand multi gravida
c. Elderly primi para d. Elderly primi gravida
Ans : d

89. A study type concerned with lived experience and is in approach to learning about what
people life experience are
a. Epidemiological b. Experimental
c.Quasi experimental d. Phenomenological
Ans : d

90. organization of contents in qualitative studies uses the following format


a. DIMRA b. IMRAD
c.MIRAD d. AIMRD
Ans : b

91. Gender, age class, religion, type of disease and blood group are measured on
a. Nominal scale of measurement
b. Ordinal scale of measurement
c. Interval scale of measurement
d. Ratio scale of measurement
Ans: a
92. The formula E (O – E)2 is used to calculate E
a. T- Test b. Chi – square
c. Correlation coefficient d. Standard deviation
Ans: b
93. IRB is an acronym for which of the following ?
a. Internal Review Board b. Institutional Review Board
c. Institutional Rating Board d. Internal Request Board

Ans: b
94. Selection of samples from every Kth case from a list is called
a. Stratified sampling b. Cluster sampling
c. systematic sampling d. Purposive sampling
Ans: c
95. The primary muscles of inspiration
a. abdominals and external intercostal
b. Diaphragm and external intercostal
c. Abdominals and internal intercostal
d. Diaphragm and internal intercostal
Ans: b
96. Clotting can be prevented by using
a. Antiemitics b. Antibiotics
c. Anticoagulants d. Analgesics
Ans: c
97. Which arrangement of gastrointestinal layer is correct from inside out?
a. Serous, Mucosa, Sub mucosa and muscular layer
b. Mucosa, Sub mucosa, Serous and muscular layer
c. Mucosa, Sub mucosa, muscular and Serous
d. Muscular, Serous, , Sub mucosa and mucosa
Ans: c
98. Match the following
(a). Pleural cavity filled with blood 1. Pneumothorax
(b). Pleural cavity filled with pus 2. Pleural effusion
(c). Pleural cavity filled with air 3. Heamothorax
(d). Pleural cavity filled with fluid 4. Pythorax
Ans: b
99. Duration of one cardiac cycle in a normal person at rest is
a. 0.3 Sec b. 0.5 Sec
c. 0.8 Sec d. 0.10 Sec
Ans: c
100. The destruction of donor red cells by and D antibodies formed from previous exposure
is called
a. Rh incompatibility b. Hemolytic anemia
c. Erythroblastosis fetals d. Agglutination reaction in coomb’s test
Ans: a
101. Which one of the following is a functionally active form of vitamin D ?
a. Cholecalciferol b. Ergocalciferol
c. Dehydrocholesterol d. Calcitriol
Ans: d
102. In prolonged hyperglycemia ,---------------- accumulates in the tissues, resulting in
microvascular complications
a. Ketones b. Lactic acid
c. Sorbitol d. Glucose
Ans: c
103. The inhibition of glycolysis by oxygen is known as
a. Bohr effect b. Crab tree effect
c. Leubering effect d. Pasteur effect
Ans: d
104. Azotemia is a symptom seen in
a. Diabetes mellitus b. Acute renal failure
b. PEM d. Scurvy
Ans: b
105. Which one of the following anticoagulant binds with Ca2+ and prevents blood clotting
?
a. Heparin b. Oxalate
c. Protein C d. Warfarin
Ans: b
106. Salivary amylase becomes inactive in the stomach primarily due to
a. Inactivation by low pH
b. Degradation by gastric pepsin
c. Inhibition by CI
d. Inhibition by peptides
Ans: a
107. Match list I with list II and select the correct answer using the codes given below the
list
List I List II
a. Allopurinol 1. Immuno suppression
b. 5 fluorouracil 2. AIDS
c. Didanosin 3. Gout
d. Azoythioprine 4. Cancer
(a) (b) (c) (d)
a. 2 3 4 1
b. 4 3 2 1
c. 3 4 2 1
d. 4 3 1 2
Ans: c
108. The partial or total separation of wound layers is termed as
a. Evisceration b. Dehiscence
c. Pressure sore d. Infection
Ans: b
109. Infectious hospital waste is disposed by the process of
a. Autoclaving b. Deep burial
c. Incineration d. Shredding
Ans: c
110. Obesity places patient at an increased surgical risk because of which of the following
factors ?
a. Risk of bleeding is increased
b. Ventilatory capacity is reduced
c. Fatty tissue has a poor blood supply
d. Metabolic demands are increased
Ans: b
111. Lateral movement of the eyeball is controlled by the following nerve
a. Auditory b. Optic
c. Facial d. Abducens
Ans: d
112. The assessment of wound indicates healing is occurring when
a. The Centre tissue is white
b. Bleeding has stopped
c. There is no further drainage from wound
d. Pink granulation tissue is visible
Ans: d
113. Massage promote increased circulation and softening of connective tissues. It also has
which effect ?
a. Narrows blood vessels b. Eases muscle spasms
c. Causes hyper ventilation d. Widens blood vessels
Ans: b
114. Which type of wound close by primary intention ?
a. Second degree burn b. Pressure ulcer
c. Traumatic injury d. Observation
Ans: d
115. Rorschach ink blot test, thematic apperception test are based on the phenomenon of
a. Projection b. Identification
c. Transference d. Observation
Ans: a
116. Individuals rate attitudes using a pair of opposite adjective in the
a. Thurstone scale b. Semantic differential scale
c. Interview d. Likert scale
Ans: b
117. Which of the following is an example of a primary prevention programme in mental
health?
a. Efforts to make the disorders less debilitating
b. Improving the study skills of school drop out
c. Genetic Counselling
d. Telephone counseling for those contemplating suicide
Ans: c
118. Consider the following stages of personality by freud
a. Oral b. Anal
3. Latency 4. Phallic
5. Genital
Ans: c
119. The drug of choice for treatment of leptospirosis is :
a. Gentamycin and kanamycin
b. Sulfonamides with cotrimaxazole
c. Penicillin and tetracycline
d. Tobramycin and cephalosporin
Ans: c
120.An Immunoglobulin which crosses the placenta is :
I. Ig G II. Ig E
III . Ig M IV. IgA
a. I and III b. I Only
c.I and IV d. II and IV
Ans: b
121. Which of the following statement is not true ?
a. Transistors and much smaller
b. Transistors produce low heat
c. Transistors were fast in conducting electricity
d. Transistors were used in ratio and other element
Ans: d
122. The concept used for father – dominated family
a. Patrilocal family b. Patriarchal family
c.Pathrilineal family d. Conjugal family
Ans: b
123. Find the odd statement
1. Casual, conventional, expressive
2. The contagion theory, the emergent theory
3. Mob, crowd , Public
4. Active crowd , audience , public opinion
a. 3 and 4 are odd statements
b. 1 and 2 are odd statements
c. 2 and 4 are odd statements
d. Only 2 odd statement
Ans: d
124. Sociology is important for nursing practice because nurse are, except
a. Critically involved in making sense and giving purpose to life , illness and
death
b. Able to discriminate people by culture
c. Confronting problems which is controversial in society
d. Engaged in the study of human societies
Ans:b
125. The form of marriage in which one man marries more than one women at a given
time
a. Fraternal polyandry b. Polyantry
c.Polygyny d. Monogamy
Ans: c
126. The causes of population growth are except
a. Widening gap between birth and death rates
b. Low age at marriage
c. High literacy
d. Religious attitude towards family planning
Ans: c
127. Benign skin tumors included the following
1. Warts 2. Angiomas
3.Kaposi’s sarcoma 4. Keloids
a. 1,2 and 3 b. 2,3 and 4
c. 1,2 and 4 d. 1,2,3, and 4
Ans: c
128. ‘Barrel chest’ is the manifestation found in the following form of chronic airflow
limitations
I. Chronic obstructive bronchitis
II. Bronchial asthma
III. Bronchitis
IV. Emphysema
a. I b. II
c.III d. IV
Ans: d
129. Inflammatory disease of the skin characterized by well define erythematous plaques
and salivary scales is
a. Psoriasis b. Acne vulgaris
c.pemphigus d. Rosacea
Ans: a
130. Which is the microvascular complication if diabetes mellitus ?
a. Diabetic nephropathy b. Coronary artery disease
c.Peripheral d. Cerebrovascular disease
Ans: a
131. Which of these is a sign of left sided heart failure ?
a. Hepatomegaly b. Jugular vein distortion
c.weight gain in a short period d. Bibasilar crackles
Ans: d
132. When planning health teaching for a patient with hiatus hernia the nurse should be
share to include instruction to
a. Eat three large meals per day with no restrictions
b. Lie down immediately following a meal
c. Eat frequent, small blend meals with high fiber content
d. Eat frequent, small soft solids with low fiber content
Ans: c
133. In the postoperative period following a hip replacement, the patient should perform
which of the following exercises to prevent DVT
I. Leg raising
II. Dorsiflexion and extension of the foot
III. Flexion and extension of the knee join
IV. Quadriceps – soothing
a. I b. II
c.III d. IV
Ans: a
134. What is the priority teaching during thoracentesis?
a. Not moving or coughing
b. Observe for changes in skin
c. Lying on the abdomen for 48 to 72 hours
d. Checking the pulse
Ans: a
135. Baroreceptors are located in the
1. Right atrium 2. Carotid artery
2. Left atrium 4. Aortic arch
a. 1,2,3 b. 2,3,4
c.1,3,4 d. 1,2,3,4
136. Match list I with list II and select the correct answer using the codes given below the
lists
List I List II
a. Rovsing’s sign 1. Pain elicited in the calf
When the Foot is dorsiflexed
b. Psoas sign 2. Pain that occurs with passive
Internal rotation of the flexed right
Thigh with the patient on supine
c. Obturator sign 3. Pain elicited by palpation of left
Lower quadrant, causes pain in the
Right lower quadrant
d. Homan’s sign 4. Pain that occurs upon slow
Extension of right thigh with the pt.
Lying on the left side
(a) (b) (c) (d)
a. 3 4 2 1
b. 4 2 1 3
c. 2 1 3 4
d. 1 5 4 2
Ans: a
137. Match
List I List II
a. Levodapa 1. Sulfide metabolite
b. Sulindac 2. 5-amino salicyclic acid
c. Dipivefrine 3. Dopamine
d. Sulfasalazine 4. Epinephrine
(a) (b) (c) (d)
a. 3 4 2 1
b. 4 2 1 3
c. 2 1 3 4
d. 1 5 4 2
Ans: a
138. The most suitable laxative for a patient with spastic constipation is
a. Dietary fibre b. Bisacodyl
c.Castor Oil d. Phenolphthalein
Ans: a
139. Which drug is preferred for prophylactic therapy of mild to moderate asthma?
a. Montelukast b. Ipratropium bromide
c. Theophylline d. Formoterol
Ans : a

140. Reye’s syndrome is the complication of


a. Polio myelitis d. Chickenpox
c.Pertusis d. Diphtheria
Ans : b

141. AIDS can be confirmed by


a. Antigen detection test b. ELISA test
c. Kampass test d. Western blot test
Ans : d

142. Very wide gap between the birth and death rates is called as
a. Life expectance gap b. Age gap
c. Population gap d. Demographic gap
Ans : d

143. The census count is taken only once in ………. Years


a. 5 b. 10
c. 15 d. 20
Ans : d

144. The most useful therapy in treating children with psychiatric disorder is
a. Behavior therapy b. Psycho therapy
c. Cognitive therapy d. Pharmacotherapy
Ans : a

145. Among the following which is matched correctly


1. Acute Illness - Short term symptoms
2. Etiology - Process of changes
3. Adaption - Health maintenance
4. Holistic health - Self care
a. 1 b. 2
c. 3 d. 4
Ans : a

146. Among the following which is matched wrongly


a. Domestic refuse -Vegetable peelings
b. Street refuse - Markets
c. Stable refuse - Metal scrapping
d. Constructional refuse - Stone pieces
Ans : c

147. Point out the wrong option in the following Indicators helping for monitoring progress
towards health for all
a. Health policy indicators
b. Social and economic indicators
c. Health status indicators
d. Environmental indicators
Ans : d

148. What is the housing standards prescription of floor space area for one person?
a. 100 sq ft b. 150 sq ft
c. 200sq ft d. 250 sq ft
Ans : a

149. Measles is otherwise called as


a. Rubeola b. Rubella
c. Small pox d. Varicella
Ans : a

150. Three tier structure of rural local government in India is


a. Community development block
b. Taluks
c. Town
d. Panchayat raj
And : d

151. The specific instructions that has to be followed by the community health nurse
during “Home visit” is called as
a. Prescription b. Job description
c. Standing orders d. Procedure manual
Ans : c

152. Mid arm circumference is measured in the age group of …….. children
a. 0-1 years b. 1-5 years
c. 5-10 years d. 10-15 years
Ans : b

153. Spot the incorrect statement regarding principles of learning according to JTFSL (Joint
Task force on student learning)
a. Learning is an active process
b. Learning is an outcome of stimulation and motivation of the learner
c. Learning is an act of self-awareness
d. Learning is an act of self-identification
Ans : d

154. Which of the following urinary stone is formed by the bacterial enzyme, urease?
a. Uric acid stone b. Calcium stone
c. Oxalate stone d. Struvite stone
Ans : d

155. Identify the odd one out


a. Morphine b. Phenobarbital
c. Noradrenaline d. Fentanyl
Ans : c
156. A burns victims with 90% burns in a multistoried shopping complex fire accident is
categorized to the following code is triageing
a. Black b. Red
c. Yellow d. Orange
Ans : a

157. The cardinal signs of Parkinson’s disease are


a. Tremor, bradykinesia and muscles rigidity
b. Paresis, anesthesia and muscles spasticity
c. Chorea and muscles hypotonia
d. Tremor, ataxia and muscles hypotonia
Ans : a

158. A man who met with a motor vehicle accidents reveals bone fragments in the X-ray of
femur. What type of fracture it is
a. Avulsion b. Compressed
c. Comminuted d. Open
Ans : c

159. Which of the following orders should a nurse question if a client is on radiation
therapy?
a. Analgesics before meals b. Saline rinses every 2 hours
c. Aspirin every 4 hours d. Balanced diet
Ans : c

160. A patient has sustained burns to anterior chest and anterior abdomen. Using rule of
nine how much of patients body surface area ha sbeen affected?
a. 8% b. 18%
c. 27% d. 36%
Ans : b

161. A 30 years old lady with normal systolic blood pressure, increased heart rate, slightly
delayed capillary refill and cool, pale skin. What is the stage of shock is the lady in?
a. Decompensated stage b. Compensatory stage
c. Hypotensive stage d. Progressive stage
Ans : b

162. A client taking cyclophosphamids (cytoxan) for treatment of lymphoma. The nurse
should be very cautions cautious in administering the medication because this drug pores
the fatal side effect of
a. Alopacia
b. Myeloma
c. Central nervous system toxicity
d. Haemorrhagic cyst
Ans : d
163. An infant weighing 5 kgs is to receive 750 mg of an antibiotic 3 times (tid) in a 24 hour
period . The available antibiotic concentration is 125 mg per 5 ml. How many ml would the
nurse administer with each dose ?
a. 10 ml b. 15 ml
c. 20 ml d. 30 ml
Ans: d
164. Match the following and write the code given as below
a. Typhoid fever 1. Pharyngeal paralysis
b. Whooping cough 2. Intestinal perforation
c. Diphtheria 3. Atelectasis
d. Mumps 4. Deafness
(a) (b) (c) (d)
a. 3 2 1 4
b. 2 3 4 1
c. 2 4 3 1
d. 2 3 1 4
Ans: d
165. Thrombocytopenia means
a. Decreased number of platelet
b. Increased number of platelets
c. Decreased number of leukocytes
d. Increased number of leukocytes
Ans: a
166. Grasping of object by closing fingers around it is by
a. Gagging reflex b. Palmer grasp
c. Rooting reflex d. Doll’s eye
Ans: b
167. According to national immunization schedule measles vaccine should be given at the
age of
a. 3 months b. 1 year
c. 2 year d. 9 month
Ans: d
168. Point out the incorrect statement
a. Growth and development is continuous and orderly
b. Growth and development is complex yet predictable
c. Growth and development is steady in rate at all stages
d. Growth and development is unique and differs from individual to individual
Ans: c
169. The milk that is secreted at the starting of regular breast feeding that contains more
protein and sugar is known as
a. Mature milk b. Fore milk
c. Hind Milk d. Transitional milk
Ans: b
170. The clinical features of an infant with hydrocephalus are
1. Bulging tensed anterior fontanelle
2. Setting – sun sign
3. Macewer sign
4. Closed skull sutures
a. 1,2 and 3 b. 2,3 and 4
c. 1,3 and 4 d. 2,4 and 1
Ans: a
171. Daily requirement of calcium for an infant is
a. 800 mg b. 100 mg
c. 500mg d. 200 mg
Ans: c
172. The process of functional and physiological maturation of the individual is known as
a. Growth c. Maturation
c. Intelligence d. Development
Ans : d

173. During the course of resuscitation, of the following responses shows effectiveness of
ventilation in the child
1. Adequacy of chest rise
2. Improved heart rate and pulse oximetry
3. Presence of increased abdominal rise
4. Improved color
a. Only 1 and 2 b. 1,2, and 4
c. Only 3 and 4 d. 1,2,3 and 4
Ans : b

174. Full thickness ( third degree)burns involves


a. 1,2,4 b.2,3,4
c. 1,3,4 d. 1,2,3
Ans : b
175. Malignant tumor of bone arising in medullary tissue is called
a. Osteosarcoma b. Ewing’s tumor
c. Osteomylitis d. Genu valgum
Ans : b

176. A positive reinforce


a. Increased he probability that a behavior will recur
b. decreases probability that a behavior will recur
c. Always results in positive behavior
d. Has nothing to do with modifying behavior
Ans : a

177. Ms :’M’ is diagnosed with an obsessive compulsive disorder. She is consistently late for
duty as a result of repetitively checking whether appliances are unplugged before leaving
home. The most appropriate nursing diagnosis for Ms. ‘M ‘ is:
a. Anxiety
b. Social isolation
c. Situational low self esteem
d. Deficient knowledge
Ans : a

178. Patient :” I cannot study. My mind keeps wandering” Nurse: “You have trouble
concentrating “ The above statements are example of the following therapeutic
communication technique:
a. Focusing b. Restating
c. Exploring d. Reflecting
Ans : b

179. An adult with major depression complains of severe sleep disturbances, daytime
drowsiness and fatigability. How would the nurse classify this sleep problem?
a. Dyssomnia b. Parasomnia
c.Secondary sleep disorder d. Primary sleep disorder
Ans : c

180. A client says to the nurse, “Life doesn’t have any joy in it anymore Things I once did for
pleasure aren’t fun”. How would the nurse document this complaint?
a. Dysthymia b. Anhedonia
c. Euphoria d. Psychomotor retardation
Ans : b
181. The drugs used used in schizophrenia are:
1. Tab.Olanzapine 2. Tab.Risperidone
3. Tab. Imipramine 4. Tab Clozapine
Choose the correct code:
a. 1,2,and 3 only b. 1,2 and 4 only
c.3,2 and 1 only d. 3,4 and 2 only
Ans : b

182. The binocular instrument providing a magnification to map the abnormal areas on
the cervix is:
a. Colposcope b. Cystoscope
c. Colonoscope d. Proctoscope
Ans : a

183. Death of a women while pregnant or within 42 days of the termination of


pregnancy irrespective of the duration and the site of pregnancy is:
a. MMR b. NMR
c. MNR d. MRR
Ans : a

184. When fetal kidney starts to produce urine?


a. 10th week onwards b. 12th week onwards
c. 13th week onwards d. 14th week onwards
Ans : a

185. The time required for a spermatogonium to develop into a mature spermatozoon
in about
a. 41 days b. 51 days
c. 61 days d. 71 days
Ans : c

186. The best contraceptive method to be adopted within 71 hours by a women who
have had unprotected intercourse is
a. IUCD b. Morning – after pill
b. Norplant d. Vaginal contraceptives
Ans : b

187. The best procedure to remove the embedded IUCD in the uterus with missing
thread is
a. Hysteroscopy b. Laproscopy
c. Cystoscopy d. Laprotomy
Ans : a

188. The term “Phlegmasia alba dolens” refers to


a. Deep vein thrombosis
b. Varicose vein
c. Retrograde extension of pelvic thrombophlebitis
d. Venous thromboembolism
Ans : c

189. Carry over effects are related to


a. Factorial design b. Pre test-post test design
c. Solomon four groups design d. Cross over design
Ans : d
190. Which of the following is least likely to be a primary source?
a. A text book
b. An article in a professional journal
c. A dissertation
d. A paper presented at a professional conference
Ans : a
191. Which one of the following is an activity of staff development except?
a. Job orientation b. Induction training
c. Continuing education d. Promotion
Ans : d

192. Which one of the following is a element of organizational behavior except?


a. People b. Environment
c. Technical d. Outcome
Ans : d

193. The type of written report used in clinical practice is:


a. Intake and output chart b. Interdepartmental report
c. Sponge book d. Dispatch index ledger
Ans : b

194. Anemia which is caused by lack of absorption of absorption of vitamin B12 due to
lack of intrinsic factor is:
a. Addisonian pernicious anemia
b. Dimorphic anemia
c. Aplastic anemia
d. Sickle cell anemia
Ans : a

195. The encysted blood so collected in the pouch of douglas is called


a. Intra ligamentary hematoma
b. Diffuse intraperitoneal hemorrhage
c. Secondary abdominal pregnancy
d. Pelvic hematocele
Ans : d

196. When applying forceps for a primigravida mother the traction (pull) required is
estimated to be about
a. 13 kg b. 18 kg
c. 14 kg d. 16 kg
Ans : b

197. Funis is otherwise called as:


a. Umbilical artery b. Umbilical vein
c. Umbilical cord d. Umbilicus
Ans : c

198. The formula used to calculate the estimated fetal weight is:
a. Naegle’s formula b. Johnson’s formula
c. Nelson’s formula d. Jacobson’s formula
Ans : b

199. The length of the fallopian tube is


a. 10.5 cm b. 9 cm
c. 11 cm d. 10 cm
Ans :d

200. Fertilization almost always, occurs in the


a. Uterus
b. Uterine tube
c. Ampullary part of the uterine tube
d. Interstitial part of the uterine tube
Ans : c

Das könnte Ihnen auch gefallen